Top Banner
Contracts Outline Burton Case Book – Prof. Perez – Fall 2011 Ch. 1. The Autonomy and Security Principles Autonomy Principle: The law empowers people to make and receive enforceable promises when they communicate decisions to act or refrain from acting in some definite way in the future, subject to other principles. Security Principle: The law requires each party to a contract formation or performance to do its part to respect the other party’s reasonable expectations and reliance. Sec. 1. Promises Hawkins v. McGee (1929) “hairy hand” Expectation interest purpose of Ks/promises. Position had the K been performed. RULE: Purpose of awarding damages for breach of K is to put the plaintiff in as good a position as he would have been in had the kept his contract. R § 344 / NOTE on interests: Expectation / Reliance / Restitution Sec. 2. Promissory Agreements A. INTENTIONS of the Parties R § 201 – Whose Meaning Prevails – allows for both subj. and obj. theories R § 20 – Effect of a Misunderstanding – allows for both subj. and obj. theories Lucy v. Zehmer (1954) “high as a Georgia pine” Issue: was there mutual assent? It is not what is said, but how it is heard and reasonably understood. Dicta: Objective Theory – even if it was assumed that Z was joking, Lucy did not realize that so it would be a valid K. Reasonable person would not known joking.
88

Contracts Outline Burton Case Book CUA Law 2011

Sep 05, 2014

Download

Documents

Robert Schafer
Welcome message from author
This document is posted to help you gain knowledge. Please leave a comment to let me know what you think about it! Share it to your friends and learn new things together.
Transcript
Page 1: Contracts Outline Burton Case Book CUA Law 2011

Contracts OutlineBurton Case Book – Prof. Perez – Fall 2011

Ch. 1. The Autonomy and Security PrinciplesAutonomy Principle: The law empowers people to make and receive enforceable

promises when they communicate decisions to act or refrain from acting in some definite way in the future, subject to other principles.

Security Principle: The law requires each party to a contract formation or performance to do its part to respect the other party’s reasonable expectations and reliance.

Sec. 1. Promises Hawkins v. McGee (1929) “hairy hand”

Expectation interest purpose of Ks/promises. Position had the K been performed.RULE: Purpose of awarding damages for breach of K is to put the plaintiff in as good a position as he would have been in had the kept his contract.

R § 344 / NOTE on interests: Expectation / Reliance / Restitution

Sec. 2. Promissory AgreementsA. INTENTIONS of the Parties

R § 201 – Whose Meaning Prevails – allows for both subj. and obj. theories R § 20 – Effect of a Misunderstanding – allows for both subj. and obj. theories

Lucy v. Zehmer (1954) “high as a Georgia pine” Issue: was there mutual assent? It is not what is said, but how it is heard and

reasonably understood. Dicta: Objective Theory – even if it was assumed that Z was joking, Lucy did not

realize that so it would be a valid K. Reasonable person would not known joking. Holding: Subjective Theory – court finds for Lucy – Based on theory that

Zehmer was not joking and there was a MEETING OF THE MINDS/mmutual assent.

NOTE on Subjective and Objective theories of assent – meeting of the minds (no hidden intent) vs. manifestation of mutual assent as interpreted by a reasonable person.

Embry v. Hargadine, McKittrick Dry Goods Co. (1907) “get your men out” verbal extension of Embry’s employment K? Subj. vs. Obj. assent – Objective

Theory of mutual assent. (Obj.) MANIFESTATION OF MUTUAL ASSENT Interpretation of terms to establish legitimate expectations:

o generally accepted meanings of terms used;o meaning of term according to trade or custom;o meaning parties have assigned to the term in course of dealings.

Held: It was reasonable for Embry to believe both parties intended to K.

Page 2: Contracts Outline Burton Case Book CUA Law 2011

RULE: The secret feelings, intentions, or beliefs of a party will not affect the formation of K if their words and acts indicate that they intend to enter into a binding agreement.

Oswald v. Allen (1969) “Rare Swiss Coins” – ambiguous terms within the small group of exceptional cases in which there is no sensible basis for

choosing between conflicting understandings. Allen wants to withdraw from sale. Did “Swiss coins” refer to all Swiss coins or just those in the Swiss Collection?

RULE: a K does not exist when the terms used to express an agreement are ambivalent, the parties understand the terms in different ways, and neither party should have reasonably been aware of the other party’s understanding.

NOTE on applicable law

B. OFFERS R § 24 – Offer defined R § 26 – Preliminary Negotiations (ads, invitations to offer) R § 33 – Certainty / Definiteness (no K without “reasonably certain terms) R § 35 – Offeree’s Power of Acceptance

Mesaros v. U.S. (1988) “special Liberty Coins from U.S. Mint” – offers vs. solicitations In determining when an offer has been made, it must be objectively reasonable

that the alleged offeree believe that the advertisement or solicitation was intedned as an offer. Ads are not – bc it is conceivable the offeror could be bound by Ks requiring the delivery of goods far in ecess of the advertiser’s supply.

RULE: Where one party solicits and receives an order or other expression of agreement from another, clearly specifying that there is to be no K until ratification or assent by some officer or representative of the solicitor, the solicitation itself is not an offer, it is a request for an offer.

Lefkowitz v. Great Minneapolis Surplus Store (1957) “first come first served stole” “First come first served” creates language of promise which is ordinarily lacking

in ads for the sale of goods. Combined with the statement of quantity (one). The “house rule.” Court finds that after acceptance (Lefkowitz showing up to

purchase the stole), the offeror cannot modify the advertisement/offer/RULE: A newspaper ad for the sale of an article which is clear, definite and explicit, and leaves nothing to negotiation is an offer, acceptance of which will create a binding K.

C. POWERS OF ACCEPTANCE R § 36 – Methods of Termination of Power of Acceptance

(1) a. rejection (38) or counter-offer (39) by offeree; b. lapse of time (41); c. revocation by offeror; d. death or incapacity

(2) terminated by non-occurrence of any condition of acceptance under terms of offer.

R § 41 – Lapse of Time – time specified in offer / at the end of reasonable time / offer sent by mail seasonably accepted if acceptance mailed at any time before midnight on the day on which the offer is received.

Page 3: Contracts Outline Burton Case Book CUA Law 2011

Akers v. J.B. Sedberry, Inc. (1955) “two TX engineers’ resignation” Issue: when does Power of Acceptance end? At the end of conversation if in

person/on phone.RULE: An offer made by one to another in a face-to-face conversation is deemed to continue only to the close of the conversation and cannot be accepted thereafter.

R § 38 – Rejection / R § 39 – Counter-offers R § 59 – Purported Acceptance which Adds Qualifications is a counteroffer

Ardente v. Horan (1976) “bid for sale of Newport RI home / my clients are concerned” Mirror Image Rule common law Was “my clients are concerned that the following items remain with the real

estate…I would appreciate your confirming that these items are a part of the transaction” indicative of a conditional acceptance or a counteroffer?

An acceptance may be valid despite conditional language if it is clearly independent of the condition. Ardente’s was not, thus no K formed.

RULE: An acceptance which is equivocal or upon condition or with a limitation is a counteroffer and requires acceptance by the original offeror.

R § 42 – Revocation of Communication from Offeror when offeree receives the offeror’s manifestation of intention not to enter into the proposed K.

R § 43 – Indirect Communication of Revocation terminates power of acceptance when offeror takes definite action inconsistent with an intention to enter into the proposed K and offeree acquires reliable info to that effect.

R § 46 – Revocation of General Offer – when an ad = an offer, pwr of acceptance is terminated when a notice of termination is given publicity by ad or other general means equal to original offer and no better means is available.

R § 50 – ACCEPTANCE by Performance; by Promise R § 53 – Acceptance by Performance; Manifestation of Intent Not to Accept

(1) An offer can be accepted by the rendering of a performance only if the offer invites such an acceptance. (2) Except as stated in §69 (acceptance by silence), the rendering of a performance does not constitute an acceptance if within a reasonable time the offeree exercises reasonable diligence to notify the offeror of non-acceptance. (3) Where an offer of a promise invites acceptance by performance and does not invite a promissory acceptance, the rendering of the invited performance does not constitute an acceptance if before the offeror performs his promise the offeree manifests an intention not to accept.

Unilateral K – a K in which only one party makes a promise or undertakes a performance; a K in which no promisor receives a promise as consideration for the promise given. (Brooklyn Bridge hypo)

Petterson v. Pattberg (1928) “mortgage repayment on Brooklyn real estate.” OFFEROR IS MASTER OF THE OFFER

Page 4: Contracts Outline Burton Case Book CUA Law 2011

Did Petterson know Pattberg sold the mortgage already? Could he claim he’d already accepted once at Pattberg’s doorstep?

Dissent: ’s promise was a snare and delusion! BUT: no duty of good faith in bargaining.

RULE: An offer to enter into a unilateral K may be withdrawn at any time prior to performance of the act requested to be done.

R § 25 – Option Ks OPTION K - a contractual obligation to keep an offer open for a specified

period, so that the offeror cannot revoke the offer during that period. The option is valid because it is supported by consideration. A K pursuant to which a seller agrees that property will be available for the buyer to purchase at a specified price and within a certain time period.

R § 37 – Termination of Pwr of Acceptance under Option K – pwr of acceptance is not terminated unless the requirements are met for discharge of k duty

R § 45 – Option K Created by Part Performance – (1) Where an offer invites an offeree to accept by rendering a performance and does not invite a promissory acceptance, an option contract is created when the offeree tenders or begins the invited performance or tenders a beginning of it. (2) The offeror’s duty of performance under any option contract so created is conditional on completion or tender of the invited performance in accordance with the terms of the offer.Comments: The rule of this section is designed to protect the offeree in legally justifiable

reliance in the offeror’s promise. Beginning preparations, though they may be essential to carrying out the

contract or to accepting the offer, is not enough. Preparations to perform may justify reliance under § 87(2). In many cases what is invited depends on what is a reasonable mode of

acceptance, see § 30. The distinction between preparing for performance and beginning performance may turn on many factors: the extent to which the offeree’s conduct is clearly referable to the offer, the definite and substantial character of that conduct, and the extent to which it is of actual or prospective benefit to the offeror rather than the offeree, as well as the terms of the communications between the parties, their prior course of dealing, and any relevant usages of trade.

R § 54 - Acceptance by Performance; Necessity of Notification to Offeror. (1) Where an offer invites an offeree to accept by rendering a performance no notification is necessary to make such an acceptance effective unless the offer requests such a notification. (2) If an offeree who accepts by rendering a performance has reason to know that the offeror has no adequate means of learning of the performance with reasonable promptness and certainty, the contractual duty of the offeror is discharged unless (a) the offeree exercises reasonable diligence to notify the offeror of acceptance, or (b) the offeror learns of the performance within reasonable time, or (c) the offer indicates that notification of acceptance is not required.

Page 5: Contracts Outline Burton Case Book CUA Law 2011

Marchiondo v. Scheck (1967) “broker obtained acceptance after offer expired” Scheck offered to sell realty to a specified buyer and agreed to pay Marchiondo a

broker’s commission. Later, Scheck revoked the offer. Shortly after the revocation and within the time limit wet by the offer, Marchiondo obtained the offeree’s acceptance.

Where an offer invites an offeree to accept by rendering a performance (unilateral K), an option contract is created when the offeree begins to partially perform. The offeror’s duty of performance under an option K is conditional on the offeree’s completion of performance. What constitutes partial performance will vary case to case.

Offeror cannot revoke offer once partial performance begun. RULE: Where an offer invites an offeree to accept by rendering a perfomance, an option K so created is conditional on the offeree’s completion of performance in accordance with the terms of the offer.

D. ACCEPTANCE

§50: ACCEPTANCE of Offer; Acceptance by Performance; by Promise. §54: Acceptance by Performance; Necessity of Notification to Offeror – no

notification of acceptance is required unless offeree has reason to know offeror has no means of learning of the performance.

§56: Acceptance by Promise; Necessity of Notification to Offeror – Except as stated in §69 (acceptance by silence)…it is essential to an acceptance by promise either that the offeree exercise reasonable diligence to notify the offeror of acceptance or that the offeror receive the acceptance seasonably.

§63: Time When Acceptance Takes Effect – Unless the offer provides otherwise, (a) an acceptance made in a manner and by a medium invited by an offer is operative and completes the manifestation of mutual assent as soon as put out of the offeree’s possession, without regard to whether it ever reaches the offeror; but (b) an acceptance under an option contract is not operative until received by the offeror.Comments: The rule that an acceptance is effective on dispatch has been extended in §63

to cases where an acceptance is lost or delayed in the course of transmission. Revocation of Acceptance. The fact that the offeree has power to reclaim his

acceptance from the post office or other company does not prevent the acceptance from taking effect on dispatch. Nor, in the absence or additional circumstances, does the actual recapture of the acceptance deprive it of legal effect, though as a practical matter the offeror cannot assert his rights unless he learns of them.

Davis v. Jacoby (1934) “inheriting old friend Whitehead’s estate” Did Whitehead want performance or promise to perform? – Davis accepts by

return promise, not by performance.

Page 6: Contracts Outline Burton Case Book CUA Law 2011

Presumption/default is Bilateral K, not Unilateral because bilat immediately and fully protects the expectations of both parties.

Compare UCC 2-206(1) “unless otherwise unambiguously indicated, (a) an offer to make a k shall be construed as inviting acceptance in any manner …reasonable in the circumstances; (b) an order or other offer to buy goods for prompt or current shipment shall be construed as inviting acceptance either by a prompt promise to ship or by the prompt or current shipment of goods”

RULE: In case of doubt, it is presumed that an offer invites the formation of a bilateral rather than a unilateral K.

R § 19 – Conduct as Manifestation of Assent – (3) in cases where a party’s conduct manifests assent even though he does not in fact assent, a resulting K may be voidable because of fraud, duress, mistake, or other invalidating cause.

R § 69 – Acceptance by Silence or Exercise of Dominion(1) Where an offeree fails to reply to an offer, his silence and inaction operate as an acceptance in the following cases only:

(a) Where an offeree takes the benefit of offered services with reasonable opportunity to reject them and reason to know that they were offered with the expectation of compensation.(b) Where the offeror has stated or given the offeree reason to understand that assent may be manifested by silence or inaction, and the offeree in remaining silent and inactive intends to accept the offer.(c) Where because of previous dealings or otherwise, it is reasonable that the offeree should notify the offeror if he does not intend to accept.

(2) An offeree who does any act inconsistent with the offeror’s ownership of offered property is bound in accordance with the offered terms unless they are manifestly unreasonable. But if the act is wrongful as against the offeror it is an acceptance only if ratified by him.Comments: The mere fact that an offeror states that silence will constitute acceptance does

not deprive the offeree of his privilege to remain silent without accepting. But the offeree is entitiled to rely on such statement as he chooses. The case for acceptance is strongest when the reliance is definite and substantial or when the intent to accept is objectively manifested though not communicated to the offeror.

Compare §§ 54 (notification of acceptance by performance) and 87(2) (option K). Also see §20 (misunderstanding) to understand §69 (1)(c).

Houston Dairy v. John Hancock Mutual Life Insurance “$16k deposit within 7 days” Offeror is Master of the Offer Default Rule: Silence is NOT acceptance. § 69 is the exception to the rule. Silence is not acceptance/silence was rejection as there was no way of knowing

whether the offer had been accepted or rejected. Silence as acceptance when a previous relationship exists, if silence had

previously been a form of acceptance between the parties.

Page 7: Contracts Outline Burton Case Book CUA Law 2011

RULE: a purported “acceptance” by an offeree after the time for acceptance designated by the offeror has expired constitutes a counteroffer, which must be separately accepted in a communication by the original offeror to the original offeree.

Cole-McIntyre-Norfleet Co. v. Holloway (1919) “home office clause” Cole delayed in notifying Holloway of its acceptance or rejection of a K for

perishable goods (meal). Time was of the essence. No notification of acceptance but ample opportunity to notify (weekly visits by

representative). Demonstrates importance of communicating acceptance. RULE: Delay in notification amounts to acceptance of an offer when the subject goods of the K will become unmarketable by delay.

Seaview Ass’n of Fire Island, NY, Inc. v. Williams (1987) “seven homes” Implied-in-fact K – case highlights the idea of fair play as an alternative to

consent in K cases. FP dictates that when parties voluntarily enjoy benefits from collective agreements, obligations will attach to the parties for the benefit of the community. Rawls deals well with free-riders.

Didn’t’ matter that Williams did not utilize the facilities; when he bought the homes he impliedly accepted the association requirements.

RULE: where there is knowledge that a private community homeowner’s association provides facilities and services for the benefit of the community residents, the purchase of property there may manifest acceptance of conditions of ownership, including payment for the facilities and services offered.

E. UCC FORMATION

NOTE on UCC – Article 2 applies to transactions in goods (2-102). “Goods” includes all things (including specially manufactured goods) which are moveable at the time of identification to the K for sale other than the money in which the price is to be paid, investment securities, and things in action.” (also unborn young of animals and growing crops 2-105(1)).

“This Act shall be liberally construed and applied to promote its underlying purposes and policies. The underlying purposes and policies of the act are to simplify, clarify, and modernize the law governing commercial transactions; to permit continued expansion of commercial practices through custom, usage, and agreement of the parties; and to make uniform the law among the various jurisdictions…” UCC §1-102.

§ 2-204 – Formation in General(1) A contract for sale of goods may be made in any manner sufficient to show agreement, including conduct by both parties which recognizes the existence of such a contract. (note p. 95 and Sun Printing)(2) An agreement sufficient to constitute a contract for sale may be found even though the moment of its making is undetermined.

Page 8: Contracts Outline Burton Case Book CUA Law 2011

(3) Even though one or more terms are left open a contract for sale does not fail for indefiniteness if the parties have intended to make a contract and there is a reasonably certain basis for giving an appropriate remedy.

§ 2-206 – Offer and Acceptance in Formation of Contract(1) Unless otherwise unambiguously indicated by the language or circumstances

(a) an offer to make a contract shall be construed as inviting acceptance in any manner and by any medium reasonable in the circumstances;

(b) an order or other offer to buy goods for prompt or current shipment shall be construed as inviting acceptance either by a prompt promise to ship or by the prompt or current shipment of conforming or non-conforming goods, but such a shipment of non-conforming goods does not constitute an acceptance if the seller seasonably notifies the buyer that the shipment is offered only as an accommodation (counter-offer) to the buyer.(2) Where the bargain of a requested performance is a reasonable mode of acceptance an offeror who is not notified of acceptance within a reasonable time may treat the offer as having lapsed before acceptance.

Comments:1. Any reasonable manner of acceptance is allowed unless the offeror makes it explicit. 2. Either shipment or a prompt promise to ship is proper means of acceptance. This section of the UCC rejects the theory that only a single mode of acceptance is normally envisioned by an offer.3. Beginning of performance by an offeree can be an acceptance so as to bind the offeror only if followed by notice within a reasonable time. Must be unambiguous. Nothing in this section bars the common law possibility that performance begun may have an intermediate effect of temporarily barring revocation. (option Ks)

ProCD, Inc. v. Zeidenberg (1996) EASTERBROOK “shrink-wrap license” UCC 2-204(1). Also note 2-602 provides that a rejection after an opportunity to

inspect may be effective unless the buyer manifests acceptance in the manner invited by the offeror.

Transactions in which the exchange of money precedes the communication of detailed terms are common. Like buying insurance or plane tickets.

RULE: a buyer accepts goods when, after an opportunity to inspect, he fails to make an effective rejection.

Empire Machinery v. Litton Business Telephone Systems (1977) “telephone system” Home office acceptance clause – means offer containing it is really a request for

an offer. Six months after Empire signed the form (constituting an offer, valid upon

acceptance by home office) and Litton took several steps indicating acceptance, Litton claimed it never accepted. UCC 2-206(1)(b)

RULE: Where a buyer-offeror makes an offer to purchase by means of a form supplied by the seller-offeree, the buyer-offeror has the power to waive any specified manner of

Page 9: Contracts Outline Burton Case Book CUA Law 2011

acceptance in such a form, and thus the offer can be deemed accepted and ripened into a K if the seller-offeree takes actions that are directed toward the contractual obligation.

NOTE on battle of the forms – the common law mirror image rule produces a “last shot” effect, favoring the party who sends the last form before delivery, usually the seller. UCC 2-207(1) abolishes the mirror image rule for Ks governed by Article 2. Consistent with 1-102(2)(b), an agreement can be found even though the parties’ writings do not manifest assent to identical terms.

§ 2-207 – Additional Terms in Acceptance of Confirmation(1) A definite and seasonable expression of acceptance or a written confirmation which is sent within a reasonable time operates as an acceptance even though it states terms additional to or different from those offered or agreed upon, unless acceptance is expressly made conditional on assent to the addition or different terms.(2) The additional terms (note: does this term mean additional and different or just additional) are to be construed as proposals for addition to the contract. Between merchants (key phrase) such terms become part of the contact unless:

(a) the offer expressly limits acceptance to the terms of the offer;(b) they materially alter it;(c) notification of objection to them has already been given or is given

within a reasonable time after notice of them is received.(3) Conduct by both parties which recognizes the existence of a contract is sufficient to establish a contract for sale although the writings of the parties do not otherwise establish a contract. In such case the terms of the particular contract consist of those terms on which the writings of the parties agree, together with any supplementary terms incorporated under any other provisions of this Act.

(3) is like 2-204 – ie contemplates formation without explicit acceptance

Ionics, Inc. v. Elmwood Sensors, Inc. (1997) “battle of the forms” 2-207(1) vs. (3) Mere acceptance of the goods is insufficient to infer consent to the seller’s terms

under the language of (1), nor do such terms become a part of the K under subsection (2). (3) prevails and the tems of the particular K consists of those on which the writings of the paries agree.

Purpose of 2-207 is to modify the harshness of the mirror image rule of rejection and counter-offer, giving power to the sender of last form (seller usually) to dictate the terms.

RULE: Where the terms in two forms are contradictory, each party is assumed to object to the other party’s conflicting clause. 2-207(3)

F. INCOMPLETE AGREEMENTSSun Printing v. Remington Paper & Power (1923) “refusal to deliver rolls of paper”

CARDOZO Incomplete agreement: fist four months were clearly established, remaining time

called for time and price to be determined by the parties, but not to exceed the price charged by a third party.

Page 10: Contracts Outline Burton Case Book CUA Law 2011

If just the price were left open, Sun could be deemed the holder of an option K. But both time and price = incomplete K.

When negotiations are in progress, agreements by the parties on specific points in the K before the K is agreed upon as a final whole, are not intended to have legal consequences until the whole K takes effect.

RULE: A K will be deemed invalid due to incompleteness if the agreement does not establish the length of time the terms of the agreement, such as the price, shall apply.

Arnold Palmer Golf Co. v. Fuqua Industries, Inc. (1976) “agreement to agree” Palmer and Fuqua negotiated and drafted a six page memo of intent outlining

their planned business relationship. Fuqua then unilaterally terminated transaction Agreement to agree on later definitive agreement vs. all relevant details included

RULE: A trier of fact must determine whether the parties intended to be bound when evidence – both the writing and extrinsic – supports reasonable inferences for both parties’ interpretations.

NOTE on agreements in negotiations – an agreement on a point is normally not intended or understood to have legal consequences until and unless agreement is reached on the final whole. But there is no reason in principle why an agreement made during negotiations cannot have legal effect if it is not too indefinite and the parties inted to be bound.

NOTE on GOOD FAITH IN NEGOTIATIONS – there is no general duty to negotiate in good faith. But, duress, fraud, misrepresentation, and restitution constrain the use of force and lies and avoid windfalls.

Incomplete Agreements – OBLIGATIONS OF GOOD FAITH

A/S Apothekernes v. I.M.C. Chemical “chem co. board refuse purchase / letter of intent”RULE: (1) the parties’ intentions determine whether an enforceable K comes into being during the course of negotiations, or whether some type of formalization of the agreement is required before it becomes binding.

(2) the letter of intent (preliminary agreement) created an obligation to negotiate in good faith, which prevents a party from renouncing the deal, abandoning the negotiations, or insisting on conditions that do not conform to the preliminary agreement.

Itek Corp. v. Chicago Aerial Indus., Inc. “ Letter of intent established need to negotiate in good faith Letter of intent may suffice to establish binding K – issue of fact.

RULE: an enforceable K may be formed in preliminary negotiations if that is the intention of the parties.

Sec. 3. The Requirement of a Writing – STATUTE OF FRAUDS Each statute will identify areas subject to the requirement of a writing (eg

agreements to arbitrate must be in writing based on US Arb. Act). Three main issues to consider:

Page 11: Contracts Outline Burton Case Book CUA Law 2011

o Whether the K falls within the state’s statute.o The specific conditions under which the statute is satisfiedo Legal ramifications if the statute is not satisfied

1 – does the K fall within the statute, ie must it be in writing? R § 110 forbids enforcement unless there is a writing in five categories (suretyship, land Ks, K that cannot be performed within one year, marriage Ks, and executor-administrator Ks). In addition 2-201 requires Ks for sale of goods worth more than $500 to be in writing, subject to several exceptions.

2 – ComLaw: writing must identify the parties, the subject matter, the essential terms, and be signed (§131). UCC § 2-201 requires writing evidence a sale of goods, quantity, and be signed by the party to be charged. Need not be complete or formal. Part performance may satisfy the statute (2-201(3)).

3 – consequences: no K claim or defense requiring enforcement of an oral K will succeed when the K is within the statute. A court will not raise the statute of frauds if a party does not.

Sometimes Statute of Frauds can become an instrument of fraud (ie when a client admits a K was made but asks the lawyer to plead the statute of frauds).

R § 129 – Action in Reliance; Specific Performance – can get around failure to comply with the statute of frauds if it is established in the transfer of an interest in land that the party seeking enforcement, in reasonable reliance on the K and on the continuing assent of the other party, has so changed his position that injustice can be avoided only by specific enforcement.

Chomicky v. Buttolph “lakeside property divided by a road / right of way” Oral modification to written land K held NOT enforceable. Chomicky may not rely on doctrine of part performance bc did not show evidence

supporting a valid claim that he substantially and irretrievably changed his position in reliance on the oral agreement.

RULE: Ks for the sale of land, and any proposed changes and modifications to such Ks, must be in writing to be enforceable.

Radke v. Brenon “lakeside property in Minn. divided among neighbors” K held enforceable; signed, orally accepted While the statute requires a writing, courts must also look to the intention of the

statute to avoid fraud in the sale of land by both parties. RULE: Under Minn. law, memoranda regarding the sale of land are sufficient under the Statute of Frauds when they express the consideration, are inscribed by the party by whom the sale is to be made, and state expressly or by necessary implication the parties to the K, the lands involved, and the general terms and conditions upon which the sale will be made.

2-201 (SoF); 2-209 – Modification, Rescission and WaiverCloud Corp. v. Hasbro, Inc. POSNER (2002) “Wonder World Aquarium”

UCC Statute of Frauds 2-201 applies, thus any modification had to be in writing. Hasbro’s emails and handwritten notes were enough to constitute a writing. Hasbro also failed to object to the modification within 10 day window 2-201(2)

Page 12: Contracts Outline Burton Case Book CUA Law 2011

Even if the informal writings did not satisfy 2-201 (SoF), they did constitute a waiver under 2-209(4) because Cloud reasonably relied on them to increase production. Requires an attempting modifier to show either that the other party unequivocally waived the required writing or that the attempting modifier itself reasonably relied on a waiver of the requirement. Cloud reasonably relied.

RULE: Informal writings (unsigned emails, hand-written notes) acknowledging a proposed modification of a K for the sale of goods for more than $500 satisfy the requirement of a writing under UCC 2-201 and 2-209. Statute of Frauds highly fact specific.

Ch. 2. Justification PrincipleThe law enforces promises when prima facie there are sufficient legal reasons for a

court to enforce the promise

3 bases for enforcing promises: Bargained-for-exchange (consideration) Reliance Unjust Enrichment

Sec. 1. Bargained-For ExchangeCongregation Kadimah Toras-Moshe v. DeLeo (1989) “wife’s deathbed gift”

A gratuitous promise lacks consideration and therefore no K has been entered. Justifiable detrimental reliance may constitute consideration, but here the fact that

they incorporated the $25k into their budget is insufficient to create an estoppel. Oral gratuitous promises too open to fraud, same as oral wills.

RULE: An oral promise to donate money is unenforceable if no reasonable detrimental reliance and no consideration.

R § 17 – consideration required to form a K – (2) NB §§ 82-94 special rules where no consideration required.

R § 71 – consideration defined = performance or return promise which is bargained for; (3) performance may consist of an act other than a promise, a forbearance (negative act), or creation, modification or destruction of a legal relationship.

R § 72-74 – Performance as consideration; exceptions (legal duties) R § 75-77 – Promise as consideration; exceptions R § 79 – Adequacy of Consideration; Mutuality of Obligation – no

requirement of mutuality of obligation or equivalence in value, as long as it is bargained for and not merely nominal.

Schnell v. Nell (1861) “wife Schnell left $ in will to Nell” Performance must be bargained for to be consideration. A nominal one cent is

insufficient, as is forbearance of a legally groundless claim. Holding is an exception to the general rule that a court will not examine the

fairness of the bargained-for-exchange (at law; reversed general rule at equity)

Page 13: Contracts Outline Burton Case Book CUA Law 2011

RULE: A K will be vitiated for lack of consideration where the consideration given by one party is only nominal and is intended to be only nominal.

Hamer v. Sidway (1891) “Dad Story, Sr. promised son $5k to forbear booze, smoking” Surrendering a legal right constitutes consideration if the minds of the paries

meet on the point. Devecmon is better argument / reliance is better rationale for result in Hamer than

lack of consideration, since only had the form of a bargained-for-exchange.RULE: Forbearance is valuable consideration.

Batsakis v. Demotsis (1949) “post-war Greece loan shark” K for $25 loan in return for payment of $2k! A bad bargain but she had to pay! Perhaps could have argued lack of capacity, unconscionability, fraud, duress,

undue influence, or mistake, but not inadequacy of consideration.RULE: Mere inadequacy of consideration will not void a K.

Newman & Snell’s State Bank v. Hunter (1928) “dead husband’s stock in Co.” Consideration can’t be worthless (insolvent deceased husband’s note) Use of “valuable” consideration theory to make socially just result emerge

RULE: in order for a K to be valid, valuable consideration must be exchanged.

Dyer v. National By-Products, Inc. (1986) “employee lost rt. foot” R § 74(1)(b) – Settlement of Claims exception to performance as consideration Forbearance to litigate as consideration (only when forbearance in good faith) It is in the interest of the legal system to encourage settlement and negotiation Consideration theory manipulated to rectify social justice concerns; when one

party has more power than another, valuable consideration doctrine can be used to level the playing field.

RULE: Settlement of an unfounded claim asserted in good faith constitutes valuable consideration for settlement agreements.

Lake Land Employment Group of Akron v. Columber “non-compete agmt” (2004) I agree with dissent – bad opinon Majority: an at-will employer’s forbearance from discharging an employee is

sufficient consideration to support a non-compete. Consideration: detriment to promisee (employer) or benefit to promisor

(employee) (continued employment). But agreement left employer free to fire at any time, with only benefit to employee not firing at time of signature.

Dissent: no consideration supports it bc nothing changed in underlying employment relationship: employer gave up nothing

(Bad) RULE: Continued employment alone satisfies the contractual requirement of consideration in a covenant not to compete entered into by an at-will employee who is already employed by the employer.

Wood v. Lucy, Lady Duff-Gordon (1917) “a creator of fashions” CARDOZO Did Wood give any consideration for his exclusive right to market her wares?

Page 14: Contracts Outline Burton Case Book CUA Law 2011

Implied-in-fact promise that Wood would use reasonable efforts to place Lucy’s endorsement and market her designs. The parties must have intended that to be the case because otherwise Lucy would receive nothing.

Codified in UCC 2-306(2) where an agreement for exclusive dealing in goods imposes an obligation to use best efforts by both parties unless otherwise stated.

RULE: While an express promise may be lacking, the whole writing may be filled with an obligation – an implied promise – imperfectly expressed so as to form a valid K

Levine v. Blumenthal (1936) “rents not paid in commercial lease”RULE: a promise to do what the promisor is already legally bound to do is invalid consideration and does not support a K.

R § 175 – When Duress by Threat Makes a K Voidable R § 176 – When a Threat is Improper UCC § 2-209(1) – no consideration nec. to modify K under UCC

Gross Valentino Printing v. Clarke (1983) “higher printing price, refusal after delivery” What happens when unclear whether UCC applies. Ct determines K for printed magazines (goods) rather than printing services

RULE: Under UCC § 2-209, a modification of an existing K within the UCC needs no consideration to be binding (but must be a modification in good faith per § 1-203).

R § 89 – Modification of Executory K (not fully performed K / ongoing transactions) - modification is binding if it’s fair and equitable in view of circumstances not anticipated by the parties at formationg. compare § 2-209(1).

Three ways law treats K modification doctrinally - § 73, § 2-209(1), § 89

Angel v. Murray (1974) “city trash collection increased 25%” Modification without consideration is okay – unanticipated circumstances. Taxpayers/Angel argued Maher already under legal duty to collect trash, thus new

consideration needed for increased $$. Court disagreed. UCC not applicable because not dealing with goods. But § 89(a) represents trend

away from rigid application of preexisting duty rule.RULE: Where unanticipated circumstances or conditions have occurred, the parties to a K may voluntarily increase the amount of compensation due even if no additional consideration is given.

Sec. 3. Reliance on a Promise – justification for enforcement when consideration is a necessary condition and is lacking, but justice requires enjorcement.

Devecmon v. Shaw ( 1888) “nephew send off to Europe at own expense” Consideration is sufficient when it involves detriment to the promisor, including

spending his own money in one certain way as opposed to any other way. Not important that Devecmon may have made trip anyway or benefited from it. Sufficient that he relied on his uncles express promise to his detriment (expense).

Page 15: Contracts Outline Burton Case Book CUA Law 2011

“Legal Detriment” test for sufficiency of consideration is the majority rule – plaintiff must show not only that the requested act benefited other party, but also that it was to his detriment.

Minority rule is adopted by restatement: either a showing of legal detriment by one party or a legal benefit by other party is sufficient. (distinction rarely arises.)

RULE: Sufficient consideration is present where the performing party, in reliance on the other party’s promise of repayment, has done something in a manner which he otherwise would not have been compelled to do.

PROMISSORY ESTOPPEL R § 90 – Promise Reasonably Inducing Action or Forbearance – a promise

which the promisor should reasonably expect to induce action or forbearance on the part of the promisee or a third person and which does induce such action or forbearance is binding if injustice can be avoided only by enforcement of the promise. The remedy granted for breach may be limited as justice requires. See §45 (Option K created by Part Performance) and §87(2) (option K part performance estoppel)

Feinberg v. Pfeiffer Co. Promissory Estoppel (1959) “promise to pay post retirement” No consideration for K (past services not valid for consideration), but reasonable

reliance on the promise. Not a breach of K action, an action under doctrine of PE. PE is exception to general rule that a promise cannot be enforced w/o valuable

consideration. “Reasonable detrimental reliance” takes its place. RULE: Under the doctrine of promissory estoppel, as stated in § 90, “a promise which the promisor should reasonably expect to induce action or forbearance of a definite and substantial character on the part of the promisee and which does induce such action or forbearance is binding if injustice can be avoided only by enforcement of the promise,” and it is not necessary that such a promise be given for consideration to be enforceable.

Hayes v. Plantations Steel Co. “company would “take care of him”” (1982) Illusory promise/K – a promise or K that is unenforceable for lack of

consideration because a promise by one of the parties to perform is completely within his discretion

A k must involve the exchange of present or future performance obligations. This promise made based on past performance does not fall within this situation.

PE doesn’t apply because Hayes didn’t detrimentally rely.RULE: A promise made to an employee upon the announcement of his retirement is not enforceable if the employee did not rely on it and no exchange of present or future obligations made as consideration.

R § 87 – Option K – binding if in writing, signed, w/consideration, exchange on fair terms. (2) – (DrennanStarPaving) reasonable detrimental reliance by offeree before acceptance is binding as an option K to extent necessary to avoid injustice.

UCC § 2-205 – Firm Offers – an offer, signed in writing, which gives assuances that it will be held open, is not revocable, for lack of consideration, during the

Page 16: Contracts Outline Burton Case Book CUA Law 2011

time stated or a reasonable time (not to exceed 3 months)…must be separately signed by offeror if on form supplied by offeree.

Drennan v. Star Paving Co. (1958) TRAYNOR “Sub’s mistake in accepted bid to GC” Did GC/offeree’s reliance make Sub/offeror’s offer binding/irrevocable? Y §87(2) GC Reasonable Reliance on FIRM OFFER by Sub, eg an implied promise to hold

the offer open. Problem: one party (Sub) is bound while the other party (GC) is not.

RULE: Reasonable reliance on a promise binds an offeror even if there is no other consideration.

Southern California Acoustics v. C.V. Holder (1969) TRAYNOR “sub listed mistake” No prior relationship. GC never communicated acceptance. Thus no K. Reliance on newspaper printing of it as sub was thus unreasonable. Sub bound by its firm offer but GC not bound until an official expression of

acceptance is communicated to Sub. RULE: Silence in the face of an offer is not an acceptance unless it is understood as acceptance due to relationship between the parties and previous course of dealings.

Sec 3. Unjust Enrichment – Quasi-contractual Recovery/Restitution

Sparks v. Gustafson (1988) “maintenance of dad’s building by friend” Gustafson put time and money into the place, non-gratuitously. Gratuitous to

father/friend, but not to son when actions went far beyond what any reasonable person would consider to be rendered by a family friend.

Restore to injured party any benefit conferred upon other party to restore status quo.

RULE: Unjust enrichment exists where the defendant has received a benefit from the plaintiff, which the plaintiff has not provided gratuitously, and it would be inequitable for the defendant to retain the benefit without compensating the plaintiff for its value.

Mills v. Wyman (1825) “care-taker of child v. father” Wyman/Dad’s promise to pay Mill’s for acts of kindness (promise to pay for past

performance) lacked justification for enforcement. DEFAULT RULE Old Case Law – no consideration for past performance Also 3rd party interest – good Samaritan, son, father

RULE: A moral obligation is insufficient as consideration for a promise.

R § 86. Promise for Benefit Previously Received – Unjust Enrichement – (1) a promise made in recognition of a benefit previously received by the promisor from the promisee is binding to the extent necessary to prevent injustice. (2) a promise is not binding under 1 if (a) the promisee conferred the benefit as a gift or promisor has not been unjustly enriched; or (b) to extent that its value is disproportionate to the benefit.

Webb v. McGowin (1935) “Webb saved McGowin by plunging with pine block”

Page 17: Contracts Outline Burton Case Book CUA Law 2011

Webb (P) saved the now-deceased McGowin from bodily harm/death by placing himself in harms way (jumped on the 75lb pine block!), subsequently suffering grave bodily harm himself. McGowin in return promised to compensate Webb $15/two weeks for the rest of Webb’s life. McGowin’s executors (Ds) now refuse to pay the promised compensation.

Breach of an express or implied (in-law or in-fact?) contract – ie promise made after benefit already received.

Holding: in this case, a moral obligation is a sufficient consideration to support a subsequent promise to pay because the benefit the promisor received was deemed material.

This is the EXCEPTION TO THE RULE – Mills v. Wyman is the default rule (ie in general and in most cases, moral obligation is not enough to enforce a promise)

Question from p.197 – should Dyer have been litigated on the same basis as Webb (yes). Is holding in Dyer a manipulation of consideration doctrine to protect the interests protected in Webb? (yes).

RULE: A moral obligation is a sufficient consideration to support a subsequent promise to pay only where the promisor has received a “material benefit” (defined: advantage gained that is essential to the performance of the agreement and without which the K would not have been entered into – good way of thinking about why this is the “minority rule,” since Webb likely performed without thinking of a potential benefit and would do so again…bc it’s moral).

Ch. 3. Justice PrincipleThe law refrains from enforcing promises when the prima facie justification

for enforcing the promise is overridden by considerations of justice.

3 principles of justice at play:1. Formal – like cases treated alike, people treated equally under the law.2. Corrective – correcting a wrong done by someone else (unjust enrichment, torts)3. Distributive – Socio-moral consideration of fair distribution of wealth/power

Several reasons courts will not enforce agreements:1. Domain of Freedom to K (limits placed on free-King by public policy)2. Mistake and/or Misrepresentation (boundaries of autonomy & corrective justice)3. Unconscionability (role of distributive justice - sales in low-income markets)4. Standard Form Contracts (unreasonable terms, unfair surprise and adhesion Ks)

Sec 1. Domain of Freedom of Contract R § 7 – Voidable Contracts: a valid K which may be legally voided at the option

of one of the parties. R § 12 – Capacity to Contract

(1) No one can be bound by K who has not legal capacity to incur at least voidable contractual duties….

(2) A natural person who manifests assent to a transaction has full legal capacity to incur contractual duties thereby unless he is

Page 18: Contracts Outline Burton Case Book CUA Law 2011

a. under guardianship, orb. an infant, orc. mentally ill or defective, ord. intoxicated.

R § 15 – Mental Illness or Defect(1) A person incurs only voidable contractual duties by entering into a

transaction if by reason of mental illness or defecta. he is unable to understand in a reasonable manner the nature and

consequences of the transaction, orb. he is unable to act in a reasonable manner in relation to the

transaction and the other party has reason to know of his condition (Ortelere/volitional test)

(2) Where the K is made on fair terms and the other party is without knowledge of the mental illness or defect, the power of avoidance under (1) terminates ….a court may grant relief as justice requires.

Ortelere v. Teachers’ Retirement Board (1969) BREITEL Ortelere changed her teacher’s retirement option, while mentally ill (unable to

make rational decisions) but seemingly lucid, to one granting her max benefits during life, but nothing after her death. She died within a month of making the change and borrowing the max on the fund, stranding her long-time husband (P) who quit his job to care for her; the Teachers’ Board (D) refused to undo her changes. (P) sued, arguing the legal definition of insanity was too narrow; the Board knew she was not mentally well; there was no injury either way to it (D).

Judge Breitel (Volitional Theory) agrees with (D), holds the contract may be voided based rule (below). Rule application depends on balancing competing policy considerations (stability of Ks vs protection of mentally ill). Here, the other party knew of illness; would not be injured if (P) wins; retirement system goal is to protect/aid teachers.

Dissent: (Cognitive Theory) Ortelere’s selection was reasonable – letter she sent to the board proves she knew what she was doing; points also to fact that they needed money and didn’t know she was about to die.

RULE: a K is voidable (power of avoidance) if: - one party is mentally ill; - the illness renders him incapable of acting in a reasonable manner; - the other party knows or has reason to know of the condition.

R § 178 – When a Term is Unenforceable on Grounds of Public Policy(1) …unenforceable on grounds of public policy if legislation provides that it is

unenforceable, or the interest in its enforcement is clearly outweighed in the circumstances by a public policy against enforcement of such terms.

(2) …consider: a. the parties’ justified expectations,b. any forfeiture that would result if enforcement were denied, andc. any special public interest in the enforcement of the particular term.

(3) In weighing a public policy against enforcement of a term, account is taken of

Page 19: Contracts Outline Burton Case Book CUA Law 2011

a. strength of that policy as manifested by legislation/judicial decisions,b. likelihood that a refusal to enforce will further that policy,c. seriousness of any misconduct involved/extent it was deliberate, andd. directness of the connection between that misconduct and the term.

R § 179 – Bases of Public Policies Against EnforcementA public policy against the enforcement of promises or other terms may be derived by the court from

a. legislation relevant to such a policy, orb. the need to protect some aspect of the public welfare, as is the case for

the judicial policies against, for example, restraint of trade/impairment of family relations/interference with other protected interests

In the Matter of Baby M (1988) “surrogacy” Stern (P) and Whitehead (D) entered into a K whereby Whitehead agreed to be

artificially inseminated by Stern for $10k and to give up all parental rights to the child. Post-birth, Whitehead refused to give up Baby M. Stern sued to enforce K.

Court decision based mostly on public policy/distributive justice concerns: o interest of the child should trump K lawo unequal bargaining position (both wealth and knowledge) of the parties.

Judge significantly regulating freedom to K. Custody is granted to the Sterns with visitation rights restored to Whitehead. Case raises questions of how to read a statute – literally or intent? – and whether

to decide based on statutory law or posited (“bc I said so”) law… Mills v. Wyman – law vs. morality. In Baby M – law as morality. Counter-argument: joint gains from trade are good, don’t limit autonomy to K What’s value of a child?

RULE: An otherwise valid K may be unenforceable if it violates public policy.

In Pari Delicto – “In Equal Wrong” …with two wrongdoers, we punish the one who did more wrong (thus typically “not in pari delicto”)

Sec 2. Mistakes & Misrepresentation Note: cases involve who should benefit from knowledge of information

o At common law, no necessity for good faith in arms-length transactions. Failure to disclose something advantageous to you does not void a K.

o Torts standard is fraudulent; Ks: to rescind a K, must show either fraudulent, or an innocent misrepresentation that is material

Mistake vs. misrepresentation / mistake vs. mere regret / assumption of risk

R § 153 – When Unilateral Mistake Makes a K VoidableWhere a mistake of one party (at the time of K) is made to a basic assumption on which he made the K has a material effect on the agreed exchange of performances that is adverse to him, the K is voidable by him if he does not bear the risk of the mistake under the rule stated in § 154, and

Page 20: Contracts Outline Burton Case Book CUA Law 2011

a. the effect of the mistake is such that enforcement of the contract would be unconscionable, or

b. the other party had reason to know of the mistake or his fault cased the mistake.

R § 154 – When a Party Bears the Risk of a MistakeA party bears the risk of a mistake when

a. the risk is allocated to him by agreement of the parties, orb. he is aware, at the time of K, that he has only limited knowledge …but

treats his limited knowledge as sufficient, orc. the risk is allocated to him by the court on the ground that it is

reasonable in the circumstances to do so R § 161 – When Non-disclosure is Equivalent to an Assertion

A person’s non-disclosure of a fact known to him is equivalent to an assertion that the fact does not exist in the following cases ONLY:

a. where he knows disclosure is necessary to prevent some previous assertion from being a misrepresentation or from being fraudulent or material (defined in next §).

b. where he knows disclosure would correct a mistake of the other party as to a basic assumption on which that party is making the K, and if non-disclosure amounts to a failure to act in good faith and in accordance with reasonable standards of fair dealing.

c. Where he knows that disclosure would correct a mistake of the other party as to the contents or effect of a writing.

d. Where the other person is entitled to know the fact because of a relation of trust and confidence between them.

R § 162 – When a Misrepresentation is Fraudulent or Material(1) A misrepresentation is fraudulent if the maker intends his assertion to induce a

party to manifest his assent and the makera. knows or believes that the assertion is not in accord with the facts, orb. does not have the confidence that he states or implies in the truth of the assertion, orc. knows that he does not have the basis that he states or implies for the assertion.

(2) A misrepresentation is material if it would be likely to induce a reasonable person to manifest his assent…

R § 163 – When a Misrepresentation Prevents Formation of a KIf a misrepresentation as to the character or essential terms of a proposed K induces conduct that appears to be a manifestation of assent by one who neither knows nor has reasonable opportunity to know of the character or essential terms of the proposed K, his conduct is not effective as a manifestation of assent.

R § 164 – When a Misrepresentation Makes a K Voidable(1) If a party's manifestation of assent is induced by either a fraudulent or a

material misrepresentation by the other party upon which the recipient is justified in relying, the contract is voidable by the recipient.

(2) If a party's manifestation of assent is induced by either a fraudulent or a material misrepresentation by one who is not a party to the transaction upon

Page 21: Contracts Outline Burton Case Book CUA Law 2011

which the recipient is justified in relying, the contract is voidable by the recipient, unless the other party to the transaction in good faith and without reason to know of the misrepresentation either gives value or relies materially on the transaction.

Rescission: The cancelling of an agreement and the return of the parties to their positions prior to the formation of the K (Aspen).

Stambovsky v. Ackley (1991) “haunted house sale” Stambovsky (P/buyer) contracted to buy Ackley’s (D/seller) house; upon

discovering it was allegedly haunted he attempted to rescind the K. Stambovsky properly examined the house and didn’t discover it’s reputation Ackley had publicized the house as being haunted and had opened it for a tour

after they had entered into the K. Majority: caveat emptor precludes the recovery of money damages; but equity

allows rescission of Ks where the seller conceals material facts. Ackley’s nondisclosure was an active concealment. Majority concerned with protecting buyers expectation interest, but fine line between regret and mistake/misrep.

Dissent: caveat emptor provides seller had no obligation to disclose; there was no deceptive or act or fraudulent concealment; reputation was discoverable.

RULE: Seller’s nondisclosure of facts solely within its knowledge and undiscoverable by a prudent buyer constitutes a basis for rescission of K.

R § 152 - When Mutual Mistake Makes a K Voidable(1) Where a mistake of both parties at the time a contract was made as to a basic

assumption on which the contract was made has a material effect on the agreed exchange of performances, the contract is voidable by the adversely affected party unless he bears the risk of the mistake under the rule stated in § 154.

(2) In determining whether the mistake has a material effect on the agreed exchange of performances, account is taken of any relief by way of reformation, restitution, or otherwise.

Wood v. Boynton (1885) “uncut diamond sold for $1” Wood (P) sold stone to Boynton (D) for $1; both parties believed it was a cheap

stone. Later it was discovered to be a diamond worth $700. The only reasons for rescinding a sale are that the buyer/vendee has committed

fraud in procuring the sale or there has been a mistake made by the seller/vendor in delivering an item other than the item sold.

Here – no fraud/both parties ignorant; and no mistake as to specific entity contracted for, just mistake as to value of the entity.

In arms-length transactions, each party assumes the risk of their judgment.RULE: In the absence of evidence of fraud on the part of the vendee, a mutual mistake as to the nature and value of a thing sold will not afford a basis for rescission of the K of sale.

Lenawee County Board of Health v. Messerly (1982) “(Pickles) sewage problem”

Page 22: Contracts Outline Burton Case Book CUA Law 2011

Lenawee County Board of Health (P) found defective sewage system 6 days after Pickleses purchased rental property “as is” from Messerlys (D) (system had been installed prior to Messerlys ownership). Pickleses sought rescission of K on grounds of mutual mistake.

Court holds no ground for rescission – mutual mistake cannot be used to relieve a party who has assumed the risk of loss in connection with the mistake. Both parties are “innocent” so Pickleses assume the loss because of “as is” clause.

See R§124 on when party bears the risk of mistake. Analagous to famous “Rose the II” or “Barren Cow” case (also mutual mistake –

court allowed seller to rescind K, keep cow when discovered with calf)RULE: Rescission is not always warranted in every case of mutual mistake that relates to basic assumptions of the parties upon which the contract was made and which materially affects the agreed performances of the parties; assumption of risk is also a key factor.

R § 155 - When Mutual Mistake as to a Written Expression Justifies Reformation of a K (clerical error)Where a writing that evidences or embodies an agreement in whole or in part fails to express the agreement because of a mistake of both parties as to the contents or effect of the writing, the court may at the request of a party reform the writing to express the agreement, except to the extent that rights of third parties such as good faith purchasers for value will be unfairly affected.

Elsinore Union Elementary v. Kastorff (1960) “honest clerical error by GC” “reason to know” case. Uniquely unilateral mistake. (see Star Paving) Kastorff (D/GC) submitted bid to school district (P) but made a clerical error in

tallying the total. school district granted Kastorff the K; Kastorff realized his error the next day and called and wrote the school district to rescind his bid. School voted not to release Kastorff, who refused to do the work.

School district found another GC and sued for the difference between Kastorff’s bid and cost of the work. Some evidence that (P) should have known of error.

Courts will generally grant relief for unilateral clerical mistakes. Only in cases of extreme negligence would a mistake of facts be actionable. Relief is granted if it is reasonable to assume that the offeree should have known of the mistake.

RULE: A prompt rescission due to an honest clerical error in a bid for a building K may relieve a party from an unfair and unintended bargain.

Sec. 3. Unconscionability

UCC § 1-103 – UCC should be applied “liberally;” common law should supplement UCC provisions.

UCC § 2-302 – Unconscionable Contract or Clause(1) If the court as a matter of law finds the contract or any clause of the contract

to have been unconscionable at the time it was made the court may refuse to enforce the contract, or it may enforce the remainder of the contract without the unconscionable clause, or it may so limit the application of any unconscionable clause as to avoid any unconscionable result.

Page 23: Contracts Outline Burton Case Book CUA Law 2011

(2) When it is claimed or appears to the court that the contract or any clause thereof may be unconscionable the parties shall be afforded a reasonable opportunity to present evidence as to its commercial setting, purpose and effect to aid the court in making the determination.

Williams v. Walker-Thomas Furniture Co. (WTF) (1965) SKELLY WRIGHT Williams (D) purchased on credit several items from WTF (P), a furniture store

with form/adhesion Ks including cross-collateralization and cross-default terms (reclaim all appliances after delinquent payment on any single one; spread each installment payment across all appliances so no one appliance ever paid off first).

Common law applies caveat emptor, which refuses to void a K unless fraudulent. Court looks to UCC, especially §2-302 – court should prevent overreaching in

adhesion Ks. To allow WTF to exploit Williams (D), who is poor and for whom credit hard to come by, would be unconscionable.

Dissent: majority overlooks high risk (high costs) of granting credit to the poor.RULE: Where, in light of the general commercial background of a particular case, it appears that gross inequality of bargaining power between the parties has led to the formation of a K on terms to which one party has had no meaningful choice, a court should refuse to enforce such a K on the ground that it is unconscionable.

NOTE on “Easy Credit” Consumer Transactions – gives weight to the dissent in WTF, noting the costs of providing credit and goods/services to the poor, something with considerable social value.

Toker v. Westerman (1970) “refrigerator-freezer door-to-door” Westerman (D) bought a fridge from a door-to-door salesman. Total price was

$1,229; value was closer to $350. After paying $650, Westerman refused to pay the balance.

Under UCC, unconscionability defined as situation in which there is inequality so strong, gross, and manifest that it is impossible to justify. Where a purchase price is so excessive to as to be shocking to the court = unconscionabilty.

Compare to consideration doctrine and the peppercorn. Generally, inadequacy of consideration, standing alone, is deemed to be insufficient grounds for finding a K unconscionable.

RULE: A flagrantly excessive purchase price for goods may be deemed unconscionable.

Frostifresh Corp. v. Reynsoso (1966) “fridge sold to Spanish-speakers” Frostifresh (P) and Reynoso entered into a K for a fridge negotiated in Spanish.

Frosti used distracting and deluding tactics to convince Reynoso to enter into the retail installment K, despite R’s objections that he couldn’t afford it. Total price would be $1,145; value only $350. Reynoso raised issue of fraud, Court on its own raised issue of unconscionablity.

Court finds unconscionability based on price, and fact that Reynoso never received a translation of the K. Court orders the price be the cost of the appliance only; Appellate court adjusts the amount to ensure fairness to both parties.

Page 24: Contracts Outline Burton Case Book CUA Law 2011

Courts look to relative bargaining powers of parties, substantive unfairness, and public policy in determining whether or not a K is unconscionable.

RULE: Courts have the power under UCC § 2-302 to enforce provisions of a K in order to prevent and unconscionable result.

Sec. 4. Standard Form Contracts

Federal Arbitration Act §§ 2, 3, 4 (p. 529 of the supplement book) Adhesion Contract: a K in standardized form that is prepared by one party and

offered on a “take it or leave it,” non-negotiable bases, and whose terms are so disproportionately in favor of the drafting party that courts tend to question the equality of bargaining power in reaching the agreement.

Procedural Unconscnionability: requires proof thato one party to the agreement lacked knowledge or voluntariness;o the written form contained inconspicuous print or complex legal terms; o the parties had a great disparity of bargaining power or sophistication; o and/or that one party had no opportunity to study and interpret the K

(Aspen) Substantive Unconscionability: K either contains terms unduly oppressive or

unfair to one party and/or terms which violate the subordinate party’s reasonable expectations. (Aspen)

Black’s on Procedural/Substative unconscionability: o procedural unconscionability results from improprieties in contract

formation (such as oral misrepresentations or disparities in bargaining position) rather than from the terms of the contract itself.

o substantive unconscionability results from actual contract terms that are unduly harsh, commercially unreasonable, and grossly unfair given the existing circumstances. 

o "Most cases of unconscionability involve a combination of procedural and substantive unconscionability, as it is generally agreed that if more of one is present, then less of the other is required." E. Allan Farnsworth

Note on cases in this section: WashMu v. Bailey least merits enforcement, Broemmer v. Abortion Services is the hardes/in the middle, We Care Hair v. Engen most merits enforcement.

Washington Mutual Finance Group v. Bailey (2004) “K w/illiterates” Illiterate recipients of loans and insurance (Ds) signed Ks to arbitrate any disputes

with WashMu (P); when dispute arose, Ds filed suit rather than arbitrate. P sued. Court held the arbitration agreements were not unconconscionable. Trial judge

was wrong to base his decision on procedural unconscionability. Simply because of their illiteracy, they do not necessarily lack knowledge about

the arbitration agreement. By signing, one indicates they know what is contained in the K. The illiterate knew they were illiterate. Each party responsible for his own interests.

RULE: An arbitration agreement is not unconscionable (under MS law) either if one party to the agreement is illiterate, and thus does not understand the agreement, or if the

Page 25: Contracts Outline Burton Case Book CUA Law 2011

other party knows of the illiteracy but fails to inform the illiterate party that the agreement requires arbitration.

R § 211 – Standardized Agreements(1) Except as stated in Subsection (3), where a party to an agreement signs or

otherwise manifests assent to a writing and has reason to believe that like writings are regularly used to embody terms of agreements of the same type, he adopts the writing as an integrated agreement with respect to the terms included in the writing.

(2) Such a writing is interpreted wherever reasonable as treating alike all those similarly situated, without regard to their knowledge or understanding of the standard terms of the writing.

(3) Where the other party has reason to believe that the party manifesting such assent would not do so if he knew that the writing contained a particular term, the term is not part of the agreement.

Broemmer v. Abortion Services of Phoenix Ltd. (1992) “abortion arbitration clause” Broemmer (P) (21yr old unmarried high school graduate) filed malpractice suit

against Abortion Services (D). D claimed she had to arbitrate based on form she signed when she arrived for the procedure.

Majority held adhesion Ks are enforceable unless it is unconscionable or beyond the reasonable expectations of the parties. In this case it was unreasonable to expect a high school grad in her circumstances to expect what was found in the forms she signed.

Court also held it would be unreasonable to enforce the critical provision requiring the arbitrator be an OB-Gyn when it was not a negotiated term and D failed to explain it or call attention to it.

Because the agreement fell outside Ps reasonable expectations, the court does not look at whether the K was also unconscionable.

Dissent: courts should prefer arbitration over litigation. Majority is out of sync with public policy.

Majority creates an exception to the traditional “signature/duty to read rule.” In this case the manifestation of assent that is implied in the signature is not reasoned or knowing (compare to WTF illiterati). Consent requires an understanding of the provision in question (P admitted she had no idea what arbitration was) and a reasonable opportunity to accept or decline. Even if those two criteria are present, courts may still refuse to enforce a clause if it’s unconscionable or contrary to public policy.

RULE: An adhesion K will be enforced unless it is unconscionable or beyond the reasonable expectations of the parties.

We Care Hair Development, Inc. v. Engen (1999) “franchise agreement, baseball bat” Franchise agreement with cross-default provision which made the breach of

franchise agreement a breach of the sublease. Franchisees’ (D) contract with WCH (P) contained an arbitration clause for any disputes, but their sublease K did

Page 26: Contracts Outline Burton Case Book CUA Law 2011

not. WCH Realty could terminate the sublease and evict the franchisee for any breach, of the sublease or of the franchise agreement.

Majority held the arbitration clauses could not be viewed as creating unfair surprise. Each franchisee was provided a copy which clearly disclosed the terms, and the parties are businesses (sophisticated). Thus not unconscionable.

RULE: K is unconscionable when, viewed as a whole, it’s oppressive/totally one-sided.

Ch. 4. Compensation PrincipleThe law enforces promises mainly by compensating nonbreaching parties for unavoidable, foreseeable, and reasonably certain harms caused by a breach.

Law tries give you the benefit of the bargain, ie expectation interest is most typical measure of remedy for breach.

Consequences of finding a breach will affect a court’s propensity to find one.

Sec. 1. Compensation or Punishment? R § 355 – Punitive Damages not available in breach of K

White v. Benkowski (1967) “neighbor squabble over water well” Compensatory damages vs. punitive damages Punitive never ok, regardless whether intentional or malicious. No moral

penalties may be assessed for mere breach of K. Breach of K actions only meant to place injured party in position if no breach

RULE: Damages may be awarded for inconvenience for breach of K, but no punitive damages may be awarded for breach of K.

R § 356 – Liquidated Damages & Penalties

City of Rye v. Public Serv. Mut. Ins. BREITEL (1974) “developers liquidated damages” When damages flowing from a breach of K would be difficult to ascertain, the

parties may provide for liquidated damages that reasonably approx. likely actual damages. If constitutes penalty, not enforceable. Compensation, not punishment.

Also the goal of K damages is to restore the damaged party not make better off had there been no breach.

RULE: An action on a performance bond will not stand if the bond amount is not related to actual damages. For actual damages is okay, for penalty not okay.

NOTE on Efficient Breach – EB as rationale not to penalize/punish; rationale to compensate vs. compel. Assumptions: 1) people act rationally to maximize their own wealth/welfare 2) K’ing parties operate within competitive markets, with many buyers and sellers; 3) act without compulsion on 4) full information regarding their preferences and available opportunities and 5) transaction costs are insignificant. Problems with compensation as motivation: 1) EB may reduce a promisee’s propensity to rely. 2) identifying the dollar figure that amounts to full compensation is hard. 3) efficiency vs. morality of keeping promises.

Page 27: Contracts Outline Burton Case Book CUA Law 2011

Sec. 2. Expectation Remedies Two calculable components: out-of-pocket losses in reliance on a promise and the

value of opportunities forgone in reliance on the promise. gain prevented. Remedies awarded for breach are 2 kinds, specific performance and damages ($)

A. SPECIFIC PERFORMANCE Specific performance unavailable unless money damages would be inadequate;

SP awarded at court’s discretion. In general SP available when goods are unique, regularly in land sale Ks, and rarely in personal service Ks.

R § 347 – Measure of Damages – based on expectation interest measured by (a) loss in value to him of other party’s performance, plus (b) any other loss, incidental or consequential, cause by breach, less (c) any cost or other loss that he has avoided by not having to perform.

R § 359(1) – Effect of Adequacy of Damages – SP will not be ordered if damages would be adequate to protect expectation interest of injured party

UCC § 2-716 – Buyer’s right to SP or Replevin – SP where the goods are unique or in other proper circumstances (ie where buyer can’t cover); (3) right to replevin if after reasonable effort he is unable to effect cover for such goods...[bottom line – UCC moves away from “uniqueness” to include K context.

McCallister v. Patton (1948) “sale of 1948 Ford tudor sedan and radio!” Good efficient breach case (?) No SP granted – the car was not sufficiently peculiar or unique. Measurable in $

RULE: Equity does not grant specific performance of a K for the sale of personal property if damages in an action at law afford a complete and adequate remedy.

London Bucket Co. v. Stewart (1951) “motel heating system” No SP granted – involved construction K, general rule is no SP bc damages are

adequate remedy and bc of incapacity of court to supervise the performance.RULE: Specific performance will not be granted unless the ordinary common law remedy for breach of K is an inadequate and incomplete remedy.

B. GENERAL DAMAGES

UCC – Seller’s Damages: Problems page 292 § 1-106 – Remedies to be Liberally Administered – remedies provided by UCC

must be liberally administered to the end that the aggrieved party may be put in as good a position as if the other party had fully performed, BUT neither consequential or special damages nor penal damages may be had except as specifically provided by UCC or other rule of law

§ 2-703 – Seller’s Remedies – withhold delivery; identify goods to the K; resell and recover damages; recover damages for non-acceptance; or in limited cases recover the price.

Page 28: Contracts Outline Burton Case Book CUA Law 2011

§ 2-704 – Seller’s Right to Identify Goods to K or Salvage Unfinished Goods - (1)(a) seller may identify to the K conforming goods not already identified if at the time he learned of the breach they are in his possession or control; (b) treat as the subj of resale goods which have been intended for the particular K even though those goods are unfinished; (2) where goods are unfinished a seller may in the exercise of reasonable commercial judgement for the purposes of avoiding loss complete the manufacture and wholly identify the goods to the K or cease manufacture and resell for scrap or salvage value, or other reasonable manner.

§ 2-706 – Seller’s Resale – (1) seller may resell; where the resale is made in good faith and in a commercially reasonable manner the seller may recover the difference between the resale price and the K price together with any incidental damages allowed under 2-710. (2) resale must be reasonably identified as referring to the broken K but it is not necessary that the goods be in existence or that any or all of them have been identified to the K before the breach. (3)Where the resale is at private sale (ie not an auction) the seller must give the buyer reasonable notification of his intention to resell (5) good-faith purchaser at resale takes the goods free of any rights of the original buyer even if seller fails to comply with this section; (6) seller not accountable to buyer for any resale profit.

§ 2-708 – Seller’s Damages – (1) measure of damages for non-acceptance or repudiation by buyer is the difference between the market price at the time and place for tender and the unpaid K price together with incidental damages, less expenses saved in consequence of buyer’s breach. (2) if (1) is inadequate to put seller in as good a position as performance would have done then the measure of damages is the profit (including reasonable overhead) which seller would havemade from full performance by buyer, together with any incidental damages..

§ 2-709 – Seller’s Price (ie Seller’s Specific Performance) – (1) seller may recover, together with incidental damages, the price of (a) goods accepted or of conforming goods lost or damaged within a commercially reasonable time after risk of their loss has passed to the buyer; and (b) goods identified to the K if the seller is unable to resell them reasonably (reasonable time/price); (2) When seller sues buyer for price he must hold for buyer any goods which have been identified to the K and are still in his control except that if resale becomes possible he may resell them at any time prior to the collection of judgment. Net proceeds of any such resale must be credited to the buyer, and payment of a judgment entitles him to any goods not resold. (3) even though a seller may be held not entitled to the price under this section, nevertheless he may be awarded damages for non-acceptance under preceding section (2-708)

§ 2-718 – Liquidation or Limitation of Damages – (1) allowed only at an amount reasonable in light of the anticipated or actual harm caused by the breach, difficulties of proof of loss. Terms fixing unreasonably large liquidated damages are void as penalty. (2) where seller justifiably withholds delivery of gods vecause of the buyer’s breach, buyer is entitled to restitution of any amount by which the sum of his payments exceeds …(b) in absence of specific liquidated damages terms, 20% of the value of the total performance for which the buyer is obligated under the K or $500, whichever is smaller.

Page 29: Contracts Outline Burton Case Book CUA Law 2011

Neri v. Retail Marine Corp. (1972) “sale of boat” Breach by buyer. 2-718 and 2-708(2) recovery of lost expected profits. 2-708(1) : measure of damages for repudiation/non-acceptance is difference

between market price plus incidentals at time and place of tender, and the unpaid K price.

2-708(2) : if that’s inadequate to make seller whole (put seller in same position as if the K had been performed) then measure is the lost profit on the K.

Also example of 2-718 deposit provision: allows defaulting buyer to acquire restitution for money advanced by allowing the defaulting buyer to recover that money in excess of the seller’s damages. Prevents unjust enrichment.

RULE: A seller may recover his lost profit from a sales K when the buyer defaults on the purchase if the K market differential measure of damages is inadequate to put the seller in as good a position as performance would have done.

Problems Page 298 § 2-711 – Buyer’s Remedies; Security Interest in Rejected Goods – (1)…buyer

may cancel and whether or not he has done so may in addition to recovering so much of the price as has been paid (a) “cover” and have damages under the section as to all the goods affected wither or not they have been identified to the K, or (b) recover damages for non-delivery (2) where the seller fails to deliver or repudiates the buyer may also (a) recover the goods if they have been identified to the K or (b)in a proper case obtain SP or replevy the goods…

§ 2-712 – Buyer’s “Cover”; Procurement of Substituted Goods – (1) buyer may cover by making in good faith and without unreasonable delay any reasonable purchase of goods in substitution for those due from the seller (2) buyer may recover from seller as damages the difference between the cost of cover and the K price together with any incidental or consequential damages, less expenses saved in consequence of seller’s breach (3) failure by buyer to effect cover within this section does not bar him from any other remedy.

§ 2-713 – Buyer’s Damages for Non-delivery or Repudiation – (1) measure of damages for non-delivery/repudiation by seller is difference between market price at time when buyer learned of the breach and K price plus incidental and consequential damages less expenses saved. (2) market price determined…

§ 2-714 – Buyer’s Damages for Breach in Regard to Accepted Goods – (1) where buyer has accepted goods and given notification (2-607) he may recover damages for any non-conformity of tender the loss resulting from the seller’s breach as reasonably determined (2) measure of damages for breach of warranty….(3) in a proper case incidental and consequential damages per 715

§ 2-715(1) – Buyer’s Incidental Damages – (1) incidental damages resulting from the seller’s breach include expenses reasonably incurred in inspection, receipt, transportation and care and custody of goods rightfully rejected, any commercially reasonable charges, expenses or commissions in connection with effecting cover and any other reasonable expense incident to the delay or other breach.

Fertico v. Phosphate Chemicals Export (1987) BELLACOSA / TITONE (dissent)

Page 30: Contracts Outline Burton Case Book CUA Law 2011

Breach by seller. Phosphate (D) breached a K for sale of fertilizer, but Fertico (P-buyer/seller – middle man) still obtained possession of the late delivered goods and resold them at a profit after covering for the breach.

§1-106 requires liberal administration of remedies; Late delivery of first shipment after Fertico covered is viewed as a separate transaction from the original K. That Fertico made a profit should not affect the damages awarded for the cost of cover.

Titone Dissent: had Phosphate fully performed, Fertico would not have had to cover but also would not have made the extra profit, thus Majority puts F in better position that it would have been had K been performed.

RULE: Gains made by an injured party on other transactions after the breach are not deducted from recoverable damages unless such gains resulted from the breach.

Vitex Mfg. Corp. v. Caribtex Corp. (1967) “overhead rehiring workers for canceled job” Overhead costs are not part of seller’s “costs” because constant not technically a UCC case, but consistent with §2-708(2). Expectation interests calculated one of two ways: expenses a party incurs meeting

the costs of the K in reliance on the promise of the other party; or value of opportunities not taken in reliance on the K.

RULE: Overhead should be treated as a part of gross profits and recoverable as damages and is not considered part of the seller’s costs.

R § 350 – Avoidability as a Limitation on Damages – (1) except as stated in 2, damages are not recoverable for loss that the injured party could have avoided without undue risk, burden or humiliation (2) The injured party is not precluded from recovery by the rule in 1 to the extent that he has made reasonable but unsuccessful efforts to avoid loss.

Parker v. Twentieth Century-Fox Film Corp. (1970) BURKE “Bloomer Girl” Parker sued 20th C Fox for damages resulting from 20th’s anticipatory breach of an

employment K to star in “Bloomer Girl” when it decided not to produce it, instead offering her a role in a western on the same terms.

Generally, there is some duty to mitigate damages by the nonbreaching party. Thus if an employee is offered a similar job, projected earnings would offset the damages.

But the two movies offered – a musical and a western – and the location of work all substantially different.

Efficient breach by 20th?RULE: Projected earnings from other employment opportunities only offset damages if the employment is substantially similar to that of which the employee has been deprived.

Peevyhouse v. Garland Coal & Mining Co. (1962) “strip-mining leased land” Efficient breach theory. Value rule vs. cost of performance rule. Old case law? Garland Coal Co refused to do certain restorative work on property it had leased

from Peevyhouse when it was finished with its mining operation. Diminution in value of land was only $300; cost of restoring it was $25k.

Page 31: Contracts Outline Burton Case Book CUA Law 2011

General rule: measure for damages is the reasonable cost of performance (restoration of the land). However, when a restorative provision is incidental to a K and would result in economic waste, the measure is the diminution in value of the land without restoring it. In this case it was incidental, thus award of $300

Dissent: breach was willful and in bad faith; cost of work would have been reasonably anticipated; allowing Garland to rescind negates P’s benefit of bargain

RULE: When the contract provision breached is merely incidental to the main purpose of the K, and where the economic benefit which would result by full performance is grossly disproportionate to the cost of performance, the damages which the promisee may recover are limited to the diminution in value resulting from the non-performance.

C. Limitations on damages – SPECIAL or CONSEQUENTIAL DAMAGES Special or Consequential damages compensate the nonbreaching party for losses

that are particular to the injured party’s situation, as well as general damages. Such damages might include lost profits or good will, for ex when failure to deliver causes buyer to miss a deadline. However such damages are limited by requirements that they be reasonably foreseeable and proven with reasonable certainty. In practice both limitations affect mainly claims for lost profits sought as special damages.

Lost profit – special damages – reasonably foreseeable and certain R § 352 – Uncertainty is a Limitation on Damages – evidence must permit

reasonable certainty.

Locke v. U.S. (1960) “Locke sued US for not being listed for typewriter repair Ks” Damages depend on certainty. Locke had reasonable probability of work.

RULE: Where the chance for a profit is not outweighed by a countervailing risk of loss, and where it is fairly measurable, courts should be allowed to value that lost opportunity.

Kenford Co., Inc. v. County of Erie (1986) “failure to build domed stadium” Use of expert opinion to present statistical projections of future business involved

too many variables, thus insufficient to support award of lost profit. Lost profit = very high burden of proof in this case.

RULE: To establish loss of future profits as damages fro breach of k, a party must demonstrate with certainty that such damages have been caused by the breach , and the alleged loss must be capable of proof with reasonable certainty.

R § 351 – Unforseeability and Related Limitations on Damages – UCC § 2-715(2) – Buyer’s Consequential/Special Damages – (2) (a) include

any loss resulting from general or particular requirements and needs of which the seller at the time of K’ing had reason to know and which could not reasonably be prevented by cover or otherwise; and (b) injury to person or property proximately resulting from any breach of warranty.

Hadley v. Baxendale (1854 England) “shipping broken mill shaft”

Page 32: Contracts Outline Burton Case Book CUA Law 2011

Hadley had Baxendale ship H’s mill shaft to get it fixed; B promised to deliver it in a reasonable time but didn’t know mill shut down in meantime and didn’t deliver it in a reasonable time.

Damages ought to be those arising naturally from the breach itself, and those as should have reasonably been in the contemplation of the parties at the time of K.

Hadley ought to have communicated the fact that the mill was shut down. It was not foreseeable. If foreseeable, B would have assumed the risk…

A seller be ware rule, balanced by foreseeability at the time of formation. UCC 2-715(2) allows buyer consequential damages (lost profit) resulting from

general or particular needs of which the seller had reason to know. RULE: Foreseeability marked at time of formation: the injured party may recover those damages as may reasonably be considered arising naturally from the breach itself, and, second, may recover those damages as may reasonably be supposed to have been in contemplation of the parties, at the time they made the K, as the probable result of a breach of it.

NOTE on Foreseeability and EB – in K law, marked at time of formation. Limiting damages to those foreseeable at earlier time (formation) encourages inefficient breach, since a party is best able to calculate the parties joint costs of breach at time of breach, taking into account latest circumstances.

Sec. 3. Reliance RemediesSecurity Stove v. American Ry. Express (1932) “delayed delivery”

Ordinarily, measure of damages is difference between market value of goods at time of delivery and value at time they should have been delivered. BUT, where the carrier has notice that delay will cause unusual loss, and notice was such that carrier can be presumed to have K’d with reference thereto, he is responsible for the actual damages occasioned by the delay

Not a suit for lost profits or goods; recovery of expenses – actual damages. Allowed to recover its expenses despite fact that it would have incurred them

regardless of breach, as a means of doing justice.RULE: Where a carrier has notice that a delay will cause a shipper an unusual loss, and where the notice was such that the carrier will be presumed to have K’ed with reference thereto, he is responsible for the actual damages occasioned by his delay.

NOTE on Remedies under Section 90 – following two cases contrast positions on whether expectation damages are ever recoverable for breach of a promise made enforceable by reliance (§90)

Issue: under §90/PE, do you only put person in position prior to K or also put in as good a position as if K had been performed (hypothetically, since no K under 90)?

Goodman v. Dicker (1948) “induced to start Emerson Radio franchise” Goodman was estopped from denying the existence of a K by reason of his

inducements upon which Dicker justifiably relied to his detriment. Expectation profits (lost profits) not awarded, but expenditures in reliance were

Page 33: Contracts Outline Burton Case Book CUA Law 2011

RULE: One who by his language or conduct leads another to rely to his detriment shall not subject such person to loss or injury by disappointing the expectation upon which he acted.

Walters v. Marathon Oil Co. (1981) “purchase of vacant gas station” Court of equity possesses discretionary power. Walters entered into position with

full expectation of future profits for his efforts. This case not consistent with majority rule in cases based on PE theory, which is

that the appropriate damage figure in such cases is only the losses sustained in reliance. BUT, this does seem equitable to include any lost profits where they can be calculated with fair certainty.

RULE: Lost profits (expectation profits), if ascertained with reasonable certainty, may be properly included with reliance damages where the sole theory for damages is promissory estoppel.

R § 353 – Loss Due to Emotional Disturbance – recovery for ED will be excluded unless the breach also caused bodily harm or K/breach was of a kind that serious ED was a particularly likely result.

Sullivan v. O’Connor (1973) “failed nose job” A party may recover those elements of damages which flow naturally and

foreseeably from a breach. Normally no K is made between physician/patient, no promise of cure. Generally

such cases decided in favor of physician on public policy grounds.RULE: Where an offer promises to enhance physical beauty, breach of the k would permit recovery for pain and suffering, mental distress, and a worsening of the condition.

Sec. 4. Restitution – to prevent Unjust Enrichment Availability of restitution as an alternative remedy for breach of K: unless

nonbreaching party has fully performed and the party in breach owes only money, restitution is an alternative to expectation or reliance damages whenever the breach is serious.

Oliver v. Campbell (1954) “attorney representing client in divorce” K’d at flat rate, then dumped when almost finished. Had he not K’d at all he

could have received more in quantum meruit, but was bound by K. RULE: One who has been injured by a breach of K may elect to pursue any of three remedies: 1. Proceed in quantum meruit so far as performance has been completed (restitution), 2. Keep the K alive for both parties; or 3. Treat the breach as a repudiation and sue for the profits that would have been realized through performance.

U.S. v. Algernon Blair, Inc. (1973) Promissee upon breach may elect to forego suit on K and recover only what it put

in under quantum meruit, as measured by reasonable value of the performance QM is undiminished by any loss which would have been incurred by complete

performance. K is probably a ceiling to amount recoverable under QM.

Page 34: Contracts Outline Burton Case Book CUA Law 2011

RULE: A promisee is allowed to recover in quantum meruit the value of services he gave to a defendant who breached their K irrespective of whether he would have lost money had the K been fully performed and would thus be precluded from recovering in a suit on the K.

Ch. 5. Autonomy Principle Again Interpretation:

o Identify express termso Interpret express termso Identify and interpret implied termso Distinguish between promises, conditions, and promissory conditions

Sec. 1. Identifying Express Contract TermsGianni v. R. Russel & Co., Inc. (1924) “selling soft drinks vs. tobacco”

Parol evidence rule stands between competing concerns: operates to prevent enforcement of an oral agreement that was actually made; or since a writing is by far the most reliable evidence of agreement, it serves to promote a complete integration of terms into the final K (“4 corners” approach -ie look at the paper/K)

RULE: An oral agreement falls outside the field embraced by the written one where parties, situated as the ones in the K, would naturally and normally refrain from including the oral agreement within the written one.

UCC § 2-202 – Final Written Expression: Parol Evidence R §§ 209, 210, 213, 214, 215, 216 – Integrated Agreements; Parol Evidence

Masterson v. Sine (1968) TRAYNOR “option to buy family ranch” New “natural” rule: What might naturally be made as a separate agreement. When

only part of an agreement is integrated, parol evidence rule operates to prevent extrinsic evidence from modifying the integrated terms, but not to prove elements not reduced to writing. In this case, common for family members to enter into Ks that lack complete formality.

BURKE Dissent: undermines parol evidence rule, making all agreements open to suspicion. De-incentivizes parties from getting it all on paper, and burden on Q.

RULE: Even where a writing appears complete on its face, evidence to prove the existence of a collateral agreement may be offered if the collateral agreement is such that it might naturally be made as a separate agreement by parties similarly situated.

Sec. 2. Interpreting Contract Terms Some courts follow Gianni approach, treating “the K” as the express terms

assented to by the parties. Express terms take a life of their own, independent of the parties’ subjective intentions or reasonable expectations.

Others follow the Masterson approach by treating the express terms as a window to the parties’ intentions or reasonable expectations. Intentions and expectations

Page 35: Contracts Outline Burton Case Book CUA Law 2011

constitute “the K”; express terms will not be given the meaning that neither party intended or expected. These tensions inform the following cases.

Pacific Gas & Electric v. G.W. Thomas Drayage & Rigging (1968) TRAYNOR Case strongly disapproves of the “plain meaning rule/4 corners rule” which

excludes evidence of trade usage, prior dealings of the parties, and circumstances surrounding creation of the agreement.

UCC 2-202 throws out the plain meaning rule. Traynor expands use of extrinsic evidence to show intent. (masterson)

RULE: The test of admissibility of extrinsic evidence to explain the meaning of a written instrument is not whether it appears to the court to be plain and unambiguous on its face but whether the offered evidence is relevant to prove a meaning to which the language of the instrument is reasonably susceptible.

R §§ 200-203, 206-207 – whose meaning prevails, preference of terms

Brinderson-Newberg Jt Vtre v. Pacific Erectors (1993) “erect complete picking setting” Follows PG&E, but sets limit to use of parol evidence to interpret disputed terms

– threshold test of “reasonable susceptibility”RULE: Where a dispute exists as to the meaning of a term or phrase in a K, parol evidence may be offered to explain the term; however, the written K must be reasonably susceptible to the meaning given by the parol evidence on an objective standard.

R §§ 1-205 (reasonable time/seasonableness), 2-208 (course of performance/practical construction)

Frigaliment Importing v. B.N.S. Int’l Sales Corp. (1960) FRIENDLY “chickens” In interpreting intent of the parties, court first turns to language of the K Then trade use, ordinary meaning throughout the country, circumstances

surrounding the transaction… Predates UCC but reasoned similarly

RULE: The party who seeks to interpret the terms of the K in a sense narrower than their everyday use bears the burden of persuasion to so show, and if that party fails to support its burden, it faces dismissal of its complaint.

Nanakuli Paving and Rock Co. v. Shell Oil Co., Inc. (1981) “Hawaii paving K” Implied terms win – text matters little at all (opposite from Corenswet) UCC acknowledges that an agreement is the bargain in fact which may extend

beyond written words. Certain industries use language in a peculiar way. Shell should have been on notice of the practice and trade of the Hawaii asphalt

industry which modified the express term at issue through regular course of dealing to include price protection of committed asphalt. This implied term, while modifying the writing, could reasonably have been found to be incorporated into the K.

Page 36: Contracts Outline Burton Case Book CUA Law 2011

RULE: Under the UCC, an agreement goes beyond the written words to mean the bargain of the parties in fact, as found in their language or by implication from other circumstances, including course of dealing, usage of trade, and course of performance.

Corenswet v. Amana Refrigeration (1979) “exclusive distributor of refrigerators” Express terms win. Good faith used as rationale when implying K terms, not used to strike down

express terms. No room in the express term here (either party can terminate for any reason) to imply some other agreement.

RULE: Where a K expressly provides that either party may terminate the agreement fro any reason, no obligation exists requiring there be a good faith reason for the termination.

Sec. 3. Implication of (Implied) Terms R § 204 – Supplying an omitted essential term

Spaulding v. Morse (1947) “college trust payments while son in Army” Every instrument is to be construed as to give effect to the intent of the parties Parties didn’t plan for the contingency that he be drafted. Purpose of the trust was

to provide for son’s education. Court implied a term consistent with the agreement – dad didn’t have to keep paying while son in Army.

RULE: If a K as a whole produces a conviction that a particular result was fixedly desired although not expressed by formal words, the defect may be supplied by implication and the underlying intention effectuated if sufficiently declared.

See Wood v. Lucy, Lady Duff-Gordon again Cardozo implies a term: holds that Wood promised to use “reasonable efforts.”

Without the implied term, the transaction could not have had such “business efficacy” as they must have intended it to have.

Bloor v. Falstaff Brewing Corp (1979) FRIENDLY “Falstaff and Ballantine bear” Bloor, reorganized trustee of once-successful Ballantine Brewery, sued Falstaff

for breach of Ktual obligation to use “best efforts to promote and maintain a high volume of sales” of the Ballantine brand.

Falstaff had right to give reasonable consideration to its own interest, but it breached its duty to use best efforts. Still, liquidated damages clause not enforced.

UCC § 2-306(2) provides for the imposition of an obligation on the part of both parties to an agreement for exclusive dealing in goods to use their “best efforts.” It’s a codification of judicial rule that certain Ks carried an implied obligation to use “best efforts” because they would otherwise lack mutuality of obligation.

RULE: A contractual provision obligating one to use its “best efforts” to promote and maintain a high volume of sales of a certain product is breached by a policy which emphasizes profit without fair consideration of the effect on sales volume.

NOTE on Implication – in conflict with autonomy of parties; try to imply terms objectively (Cardozo in Lady Duff states term as reasonable business ppl would in

Page 37: Contracts Outline Burton Case Book CUA Law 2011

the parties’ shoes). Autonomy is always in context. “We cannot live the life of an honored cannibal!” to quote the casebook author!

R § 205 – GOOD FAITH – [Good Faith comes between sections on interpretations.] Every K imposes upon each party a duty of good faith and fair dealing in its performance and its enforcement.

UCC § 1-201(20) and 2-103(1)(b) - “good faith” means honest in fact and observance of reasonable commercial standards of fair dealing”

Greer Properties v. LaSalle National Bank (1989) “cleanup of IL property Old Orchard” Usually, an absolute right to terminate need not comply with a standard of good

faith if the right is expressly stated in the K (see Corenswet v. Amana). Here though, termination was conditional, authorized only in the case of

“economic impracticability.” The implied good faith limits the party’s discretion to terminate.

The Sellers gave up their opportunity to shop around for a better price. Using the termination clause to recapture that opportunity was to act in bad faith.

RULE: Every K implies good faith and fair dealing. A party vested with contractual discretion must exercise that discretion reasonably

NOTE on GOOD FAITH PERFORMANCE and discretion of the parties – R § 205 comments: good faith requires faithfulness to an agreed common purpose and consistency with the justified expectations of the other party; it excludes a variety of conduct characterized as “bad faith” because violates community standards of decency, fairness or reasonableness. – Parties often want flexibility in Ks, thus add discretion into them, eg to terminate when in their “best business interest.” Courts use GF to constrain such discretion.

UCC § 2-306 – Output of the Seller, Requirements of the Buyer, or Exclusive Dealings – (1) output of seller or requirements of buyer means such actual quantity as may occur in good faith , except that unreasonably disproportionate to any estimates provided. (2) an agreement for exclusive dealings imposes an obligation by the seller to use best efforts to supply the goods and by the buyer to use best efforts to promote their sale.

Eastern Air Lines, Inc. v. Gulf Oil Corp. (1975) “fuel freighting requirements K” Eastern is good faith buyer (unlike Orange & Rockland, bad faith buyer) Price changes post embargo were not as surprising to industry, thus Gulf not

treated unfairly. (actually profited in many cases) Good faith in UCC requires honesty in fact in the conduct/transaction. But in the

case of merchants it also requires observance of reasonable commercial standards of fair dealing in the trade.

Gulf knew of fuel freighting for 30 years dealing with EA, never complained.RULE: If the established course of dealing, the established usages of the trade, and the basic K have existed without dispute for a substantial length of time, a party may not defend its breach by complaining about practices in the trade unless the other party has in fact acted in bad faith.

Page 38: Contracts Outline Burton Case Book CUA Law 2011

Orange and Rockland Utilities, Inc. v. Amerada Hess Corp. (1977) “requirements K” Bad faith buyer(/seller) (O&R) requirements K with Hess set fixed price for

buyer’s requirements. When mkt price doubled, buyer dramatically increased its “requirements” in order to resell at a profit. Robbery of Hess through speculation/arbitrage.

Buyer changed position unilaterally so as to profit from the requirements K at the seller’s expense, effecting an arbitrage on the K price, thus acting in bad faith.

RULE: A buyer in a rising market has acted in bad faith if the fixed price in a requirements K is used for speculation, or if conditions at the time of King are unilaterally changed by the buyer to take advantage of the market at the sellers expense.

NOTE on Lawful Performance of a Contract – “lawful performance doctrine” says that the law is always implied term of K. See Citizens below.

Citizens for Preservation of Waterman Lake v. Davis (1980) Terms of a K must comply with the law. But if a party violates the law in the

course of performance, the violation does not automatically place them in breach. Citizens attempting to enforce the Wetlands Act as a term of the K where they

have no cause of action under the Act itself. Can’t claim breach.RULE: While existing law is an implied term of every K, it acts only to clarify ambiguous Ks and cannot give rise to a cause of action where the law itself grants none.

Sec. 4. Promises and Conditions Terms may be viewed as implied or express Terms may also be viewed as promises, conditions, or promissory conditions.

o Promise – creates an obligationo Condition – determines when obligation to perform promise is due.

R § 224-227 – Conditions, Constructive Conditions (“implied in law” by the court);

R § 229 - Excuse of a Condition to Avoid Forfeiture – to the extent that the non-occurrence of a condition would cause disproportionate forfeiture, a court may excuse the non-occurrence of that condition unless its occurrence was a material part of the agreed exchange.

Jungmann & Co., Inc. v. Atterbury Brothers, Inc. (1928) “shipment of casein” Precedent condition to notify buyer by cable of shipment. Court strictly enforces the letter of the K, illustrating judicial deference to the

right of bargaining parties to create their own obligations by K. Substantial performance is ordinarily not applicable to excuse the nonoccurrence

of an express condition precedentRULE: A party to a K may not recover damages if the party has not performed all its conditions precedent under the K

Peacock Construction v. Modern Air Conditioning (1977) “rooftop swimming pool” Owner’s payment to the GC will not be interpreted as a condition precedent to

payment of the subs unless such is unambiguously provided for in the K.

Page 39: Contracts Outline Burton Case Book CUA Law 2011

Typically, GC, not the Sub, intended to assume risk of owner’s failure.RULE: Ambiguous provisions in subcontracts which do not expressly shift the risk of payment failure by the owner to the subK will be interpreted as constituting absolute promises to pay and not as setting payment by the owner as a condition precedent to payment.

Burger King Corp. v. Family Dining, Inc. (1977) “exclusive territorial license” Disproportionate Forfeiture and Implied Waiver BK didn’t require strict compliance with contractual term for years, then sought to

void the K for violation. BK impliedly waived its rights under the term. Waiver: intentional relinquishment of a known right (implied or express) General rule: a condition may be excused without other reason if its requirement

will involve extreme forfeiture or penalty, and its existence or occurrence forms non essential part of the exchange for the promisor’s performance. When a party indicates by its actions that it does not consider strict compliance with a condition important, it demonstrates that the condition does not form an essential part of the K.

BK waived its right and demonstrated the term not essential to the K, and to void the K would constitute extremer penalty to FD given their investment.

RULE: A party to a K may, by its actions, excuse the other from compliance with a contractual condition (thereby effectively waiving its rights under the term)

Wisconsin Knife Works v. National Metal Crafters POSNER v. EASTERBROOK (‘86) What is a waiver? UCC § 2-209 – modifications don’t require reliance or fresh

consideration. Is there a requirement of reliance with implied waivers? Election waiver = simple choice, reflecting will. Estoppel waiver = reliance, reflecting consideration substitute.

POSNER (majority): wants 2-209 to ensure consideration, focus on social utility and economic/K efficiency.

EASTERBROOK (dissent): thinks 2-209 intended to limit fraud, focus on free will (“will theory”), libertarian.

RULE:

NOTE on Conditions within a Parties Control – if my duty to perform is contingent on a condition within my control, a moral hazard is created which I can manipulate. Court prevents this by implying a duty of good faith on me. Two cases that follow involve this. Question becomes whether good faith judged subjectively (motivation for discretion) or objectively (reasonable discretion).

Fry v. George Elkins Co. (1958) “Fry’s conditions on Fry’s purchase of home” Nowhere is the obligation to act in good faith more important than where a party

has control over a condition precedent to performance. Objec: Default rule: when a party is in control of a condition to performance,

analyze behavior with an objective standard of review. It was objectively unreasonable for Fry to refuse an offered loan when he had conditioned the agreement on securing such a loan.

Page 40: Contracts Outline Burton Case Book CUA Law 2011

Subj: Moreover, there was evidence that he had changed his plans and was no longer interested in buying, thus calling his motives into question pursuant to a subjective standard of inquiry.

RULE: The failure to make a good faith effort to carry out an obligation essential to the consummation of a deal constitutes breach of K.

Pannone v. Grandmaison (1990) “CT resident highly fearful of radiation” Pannone conditioned the purchase of home on radon gas inspection. Though

within safe limits, he canceled the K. Subjective standard of good faith applied. Normally sufficiency of effort is

judged by Obj standard but where personal satisfaction is a condition, standard becomes more subj. Long history of Pannone fearing radioactivity.

RULE: If an agreed-upon contractual condition based on personal satisfaction leaves one party the discretion to approve whether the condition has been met, that party must act in accordance with a standard of subjective good faith (determined by that party’s motives)

Godburn v. Meserve (1944) “agreement to live with neighbor until death, for her house” What constitutes prevention of performance.

RULE: In order to constitute prevention of performance, the conduct on the part of the party who is alleged to have prevented performance must be wrongful, and accordingly, in excess of his legal rights.

Ch. 6. Security Principle Again Expectation interest can be broken into interest in present performance and

interest in future performance. Harm to the latter implements security principle. Interest in continuing sense of reliance and security during executory Ks is

protected by the law of breach by anticipatory repudiation. You can’t fire me – I quit!

Sec. 1. Interests of the Parties Impaired by a Breach – ie MATERIAL BREACH

R § 238 – Effect on other party’s duties of a failure to offer performance

Kingston v. Preston (1773 England) “Kingston failed to provide sufficient security” Kingston must show that he has provided or is ready to provide sufficient security

as a condition precedent to Preston’s duty to sell. Intent of parties matters too.RULE: Breach of a covenant by one party to a K relieves the other party’s obligation to perform another covenant which is dependent thereon, the performance of the first covenant being an implied condition precedent to the duty to perform the second covenant.

R §§ 250-257 – effect of prospective non-performance/repudiation UCC §§ 2-609 – 2-611 –right to adequate assurance; anticipatory repudiation

Hochster v. De La Tour (1853 England) “courier hired, dropped prior to performance” Leading case on doctrine of anticipatory breach.

Page 41: Contracts Outline Burton Case Book CUA Law 2011

Where a party manifests prospective unwillingness to perform, the other party may suspend his performance and change his position without surrendering his right to sue after the breach occurs.

Court could have considered the repudiation as (1) a defense to an action brought by the repudiating party and (2) an excuse of the constructive condition that nonbreaching party be ready, willing and able to perform per the K.

RULE: A party to a K who renounces his intention to perform may not complain if the other party, instead of waiting until performance is due, elects to sue immediately for breach of K.

U.S. v. Seacoast Gas Co. “locus poenitentiae/place of repentance – Zell owned both” Seacoast anticipatorily breached; US notified it had three days to retract the locus

poenitentiae, then altered its position before Seacoast retracted its repudiation. UCC 2-611 retraction of anticipatory repudiation - (1) the view of this case “until

the repudiating party’s next performance is due he can retract his repudiation unless the aggrieved party has, since the repudiation, canceled or materially changed his position or otherwise indicated that he considers the repudiation final”. The three day notice was indication of when repudiation was final, statement that US was relying on the repudiation.

RULE: A repudiation may be retracted up until the other party has commenced an action thereon or otherwise changed his position.

Pittsburgh-Des Moines Steel Co. v. Brookhaven Manor Water Co. (1976) “water tank” PDM asserted that it was entitled to demand assurance for the price of the tank it

was building by having the payment placed in escrow or get a personal guarantee from president of company, suspending manufacture until it happened.

UCC 2-609(1) applies – if either party has “reasonable grounds for insecurity” he can demand in writing adequate assurance of due performance, and if commercially reasonable, suspend performance.

Majority: no reasonable grounds for insecurity here. (majority). Concurrence: yes grounds for insecurity, but 2-609(1) does not give the alarmed

party a right to redraft the K and what PDM demanded would work significant changes into the K, more than a commercially “adequate assurance of due performance.”

RULE: If either party under a sales K has reasonable grounds for insecurity with respect to he performance of the other, UCC § 2-609(1) gives him the right to demand in writing adequate assurance of due performance and, if commercially reasonable, to suspend any performance for which he has not already received the agreed retrun until such assurance is received.

NOTE on Interests of Parties in Breach

Cosden Oil & Chemical Co. v. Karl O. Helm (1984) “force majeure/ frozen canal” K for sale of chemical during time when oil prices were making chemical more

expensive but a series of other unexpected problems left Cosden unable to meet

Page 42: Contracts Outline Burton Case Book CUA Law 2011

the K requirements. The longer Helm waited to act, the greater the damage figure became due to rising market price.

Generally, a buyer learns of a seller’s breach at or after the time for performance. In that case, market price at the time the buyer learns of the breach is appropriate.

But in the rare case where seller anticipatorily repudiates and the buyer does not cover, the time is less clear but determined as commercially reasonable time.

Purpose of rule is to prevent econ incentives from influencing buyers and sellers. Using commercially reasonable time as the window of opportunity, the buyer can explore cover options, the seller can attempt to perform/retract repudiation, and neither party will bear the entire burden of changing market prices.

RULE: If a buyer learns of a seller’s anticipatory repudiation before the time of performance and elects not to cover, damages may be fixed at the difference between the K price and the market price at any commercially reasonable time after the buyer learns of the repudiation.

Britton v. Turner (1834) “year-long labor K, abandoned after 9 months” Britton refused to pay Turner for the 9 months he worked since K was for a year Is the breaching party entitled to recover anything when they have willfully

breached? Yes – in order to avoid unjust enrichment, so long as it does not incentivize breach (thus so long as it is less than the K’d compensation)

RULE: Equity requires that where part of the labor of a service K has been rendered and the party has then voluntarily abandoned the K, recovery for the fair value of the labor completed may be had in quantum meruit, so long as it is less than the contracted compensation.

Sec. 2. Cancellation in Response to Breach Material Breach – triggers a power to cancel which brings the K to an end,

discharging all executory duties of both parties. A party may cancel when the other party materially breaches/does not substantially perform.

First question: is there material breach or substantial performance? What test used to determine material or immaterial breach (different test used next 4 cases)?

Second question: what measure of damages. R §§ 235, 237-239, 241 – circumstances significant in determining whether a

failure is material – list (a)-(e). 242 – circumstances significant in determining when remaining duties discharged.

Jacob & Youngs v. Kent (1921) CARDOZO “not all piping was Reading manufacture” Kent wanted plumbing replaced but it was encased in the walls. Jacob refused,

stating that it was of comparable price and quality. Omission wasn’t fraudulent or willful, just oversight. Kent refused to pay the balance of the K. Substantial performance, not material breach (key that it was not willful). Still liable for damages but in lesser amount.

Moral test: weigh the significance of the default against the oppression of the forfeiture (disproportionate forfeiture). the breach is considered to be trivial and innocent. Did not frustrate the purpose of the K.

Page 43: Contracts Outline Burton Case Book CUA Law 2011

Here, measure of damages is not cost of replacement, but variance in value btw house as specified and house as constructed. Otherwise “grossly unfair.”

Rule to promote justice when there is substantial performance with trivial deviation.

Avoids unjust enrichment which would result from award of damages to owner, who would not bother to change the piping. The unhappy buyer can return the goods or refuse delivery; the unhappy landowner keeps the incomplete structure; hence greater possibilities for unjust enrichment. Compare to PEEVYHOUSE

RULE: An omission, both trivial and innocent, will sometimes be atoned for by allowance of the resulting damage and will not always be the breach of a condition to be followed by forfeiture. For damages in consruction Ks the owner is entitled merely to the differnce between the value of the structure if built to specifications and the value it has as constructed.

Plante v. Jacobs (1960) “misplaced wall by one foot” “Essential purpose of the K” test: substantial performance is determined as where

the performance meets the essential purpose of the K. Here, the Jacobses used a standard floor plan and though they were dissatisfied, the K was substantially performed. Didn’t affect value of the home. “diminished value” rule, not “cost of repair” rule.

Measure of damages should be the K price less diminished value of home.RULE: There can be no recovery on a K as distinguished from quantum meruit unless there is substantial performance which is defined as where the performance meets the essential purpose of the K.

Walker & Co. v. Harrison (1957) “neon sign rented to dry-cleaning business/tomato!” Factors test: 1. Extent the K has been performed prior to breach; 2. Was the

breach willful; 3. Was the breach “quantitatively” serious; 4. Consequences of the determination (ie extreme hardship or disproportionate forfeiture on one of the parties). Last factor may be most important, acknowledges “materiality” as a conclusory label to be applied insofar as a sense of justice requires it. R § 242 includes whether breach done in good faith/fair dealing.

If a breach is not material, the aggrieved party may recover damages but may not cancel the K. Harrison probably alleging material breach as a way out of K.

RULE: a party attempting to repudiate a K must convince the court that the other party has materially breached the K

NOTE on the Doctrine of Material Breach – creates power of cancellation, which is like buyer’s cover w/goods.

Plotnick v. Pennsylvania Smelting & Refining Co. (1957) “seller of battery lead” Installment K - payments were late, but deliveries were often late as well. Because a buyer refuses to pay one installment, it does not automatically follow

that the seller is excused from performance on further installments. Question is whether the breach is so material that it justifies cancelling the K, or

whether it is severable, giving rise to a claim for compensation but not a right to

Page 44: Contracts Outline Burton Case Book CUA Law 2011

cancel. Factors include whether non-payment makes it impossible to deliver next installment (but P had plenty of lead) and whether non-payment creates sufficient apprehension of future non-payment and risk (but parties had long course of dealing and had always been paid in full).

RULE: Where an installment K exists and the buyer refuses to pay on one installment, it depends in each case on the terms of the K and the surrounding circumstances, whether the breach of K is so material as to justify the injured party in refusing to proceed further or whether the breach is severable, giving rise to a claim for compensation, but not a right to treat the K as broken.

McCloskey & Co. v. Minweld Steel Co. (1955) “trouble finding steel / Korean War” R 251 or UCC 2-609? Minweld Sub had trouble coming up with the steel required for it to perform, it

wrote McCloskey asking for help. The latter took the letter as an admission of breach and terminated the agreement. Determined it not to be a breach of K.

“I will not perform” is an anticipatory breach if said before performance is due. The other party may suspend performance and change position in reliance, and may sue immediately.

This case, statement was “I doubt I will be able to perform.” Not repudiation. All other party could do is suspend performance and demand assurances.

Questionable. Did court just decide not repudiation because war broke out/sympathy for minweld?

RULE: In order to give rise to a renunciation amounting to a breach of K, there must be an absolute and unequivocal refusal to perform or a distinct and positive statement of an inability to do so.

K&G Construction v. Harris (1960) “did accident = breach of ‘workmanlike manner’” Sub negligently damaged house with bulldozer, GC refused to pay installment

until damage repaired.RULE: The failure of a party to a K to perform in a substantial and negligence-free manner will justify the withholding of an installment payment, and so long as withholding was justified, the party will have wrongfully repudiated if work is abandoned.

Gill v. Johnstown Lumber (1892) “lumber delivery and the Johnstown Flood” Defining Ks as severable allows for more equitable decisions. Payment for

completed items can be rendered, and unjust enrichment is avoided. Here, the K apportioned payment.

RULE: If a K consists of several and distinct items, and the price to be paid is apportioned to each item, the K can be severed into its component parts. However, if the consideration to be paid is single and entire, the K is entire even if the subject of the K consists of distinct and independent items.

UCC §§ 2-508, 2-601, 2-602, 2-606 – 2-608, 2-612

Ramirez v. Autosport (1982) “rescission for purchase of camper”

Page 45: Contracts Outline Burton Case Book CUA Law 2011

PERFECT TENDER rule. Buyer may reject the goods based on any nonconformity prior to acceptance. However, such rejection does not terminate the K. The seller has an absolute right to cure, within the time specified for performance under the K or if after, then within a reasonable time. Where the buyer has accepted the goods, revocation is only available where the defect substantially impairs their worth. If the seller does not cure the defects, then the buyer may terminate/cancel the K.

Buyer only has right to rescind a K for the sale of goods where the nonconformity amounts to a material breach, mitigated by seller’s right to cure.

Ramirez rejected within a reasonable time; Autosport failed to cure. R’s properly canceled.

RULE: Under a K for the sale of goods, the seller is required to furnish a “perfect tender” of the subject matter of the K, and the buyer may reject any nonconforming goods.

NOTE on Performance and Enforcement – powers of rejection and revocation are conditioned on giving notice to the seller)

Sec. 3. Agreed Enforcement Terms Good faith is a condition to the valid exercise of remedial rights provided by K

(and, under the UCC, by statute) (such as “acceleration clauses” and “termination clauses”). Not as such a breach of K.

UCC § 1-208 – Option to Accelerate at Will – a term providing that one party or his successor in interest may accelerate payment or performance or require collateral or additional collateral “at will” or “when he deems himself insecure” or in words of similar import shall be construed to mean that he shall have power to do so only if he in good faith believes that the prospect of payment or performance is impaired. The burden …

Baker v. Ratzlaff (1977) “popcorn buyer vs. popcorn grower/seller” Termination clause. Good faith covers performance and enforcement aspects of a K.

RULE: The duty of good faith extends even to termination clauses, and as a result, a seller will have acted in bad faith if he abuses his discretion in utilizing a termination provision without first attempting to secure performance from the buyer.

Brown v. AVEMCO Investment Corp. (1979) “airplane used as security to a loan” Acceleration clause. Airplane used as security for a loan was going to be purchased by group of

buyers, ready and willing to pay the remainder of the debt, but AVEMCO refused to accept payment of the remaining debt and repossessed the plane.

Acceleration clauses are to protect creditor from actions by debtor which jeopardize or impair the creditor’s security. Not to be used offensively, for commercial gain, as here. AVEMCO had no claim that it had good faith belief its interests were in danger.

Better justification for result reached: the offer to pay off the mortgage could have been construed as meeting the condition precedent for release of the security

Page 46: Contracts Outline Burton Case Book CUA Law 2011

interest. Since AVEMCO refused performance, the result was that they were in breach by revocation. The security interest would thus have been lifted by the breach.

RULE: The UCC requires that options to accelerate a debt be exercised with a good faith belief that the prospect of payment or performance is impaired.

Burne v. Franklin Life Insurance Co. (1973) “insurance benefits if death 90 days later” Poorly decided - better had the court focused on unconscionability. Undermines

freedom to K and certainty of bargain. Agree with dissent.RULE: Public policy concerns mitigate against the enforcement of an insurance K term that prohibits accidental death benefits if death occurs more than ninety days after an accident.

Ch. 7. Boundaries of AutonomyAutonomy as exercised in social context and circumstances at formation. What happens when circumstances change:

Sec 1 – traditional doctrines; excuse due to changed circumstances (impossibility commlaw/impractibility UCC, but same thing)

Sec 2 – recent changes conferring greater judicial warrant to adjust or modify Sec 3 – using Tort principles to impose additional duties on the parties

Sec. 1. Excuse for Nonperformance R §§ 261-272 – Impracticability of Performance and Frustration of Purpose

Taylor v. Caldwell (1863 Blackburn) “lease for Surrey Gardens and Music Hall” Impossibility. Hall destroyed by fire before any performance. Caldwell (lessor)

excused from performance. Performance only excused if destruction of the thing is without fault. Modern view found in UCC 2-613 – for total destruction of goods the K is

avoided or the goods can be accepted with allowance for their lesser value.RULE: In Ks in which the performance depends on the continued existence of a given person or thing, a condition is implied that the impossibility of performance arising from the perishing of the person or thing shall excuse the performance

Krell v. Henry (1903) “renting London room with view of King’s coronation” Frustration of purpose. An extension of Taylor v. Caldwell. Failure of consideration or allocation of risks? Note frustration should be total or nearly total.

RULE: Where the object of one of the parties is the basis upon which both parties K, the duties of performance are constructively conditioned upon the attainment of that object.

UCC 2-613 – 2-616 – Excuse

Northern Indiana Public Service Co. v. Carbon County Coal (1986) POSNER

Page 47: Contracts Outline Burton Case Book CUA Law 2011

Force Majeure clause. NIPSCO claimed it was excused from its obligations under fixed-price K with Carbon when the state made it more expensive to use coal. (force majeure)

Purposed of fixed price Ks is to shift risk of K to buyer. Performance may only be excused when unforeseeable circumstances make performance impossible. Regulations simply made performance more expensive (relative to other cheaper source of electricity), a risk NIPSCO assumed.

RULE: Performance under a fixed-price K is not excused when circumstances cause the K to be less profitable than originally planned.

Transatlantic Financing Corp. v. U.S. (1966) SKELLY WRIGHT “wheat to Iran” Was the K legally impossible, ie only able to be done at an excessive and

unreasonable cost, when Egypt nationalized the Suez Canal forcing ship around Cape of Good Hope? No.

1st look to which party assumed the risk of unforeseen circumstances. 2nd look to see whether performance was legally impossible, ie excessive and unreasonable cost. Knowledge of crisis tends to show assumption of risk – foreseeablitty of the risk (a fiction like implied conditions).

RULE: When the issue of impossibility is raised, the court must construct a condition of performance based on changed circumstances involving the following: 1) a contingency, something unexpected, must have occurred; 2) the risk of unexpected occurrence must not have been allocated either by agreement or custom, and 3) occurrence of the contingency must have rendered performance commercially impracticable.

Sec. 2. Adjustment of Contractual Relations

Burger King v. Family Dining (again) clear from record of dealing that main focus of concern was the development of

restaurants not the literal compliance with the schedule. Intent was that the schedule be a condition subsequent and not a promise. As such, its breach did not give rise to cancellation of the K.

also, to interpret the K in a way so as to declare the schedule a promise would cause Family to suffer disproportionate forfeiture. The law of equity abhors a forfeiture.

RULE: Whether words constitute a condition or a promise is a matter of the intention of the parties to be ascertained from a reasonable construction of the language used and the surrounding circumstances.

Bak-A-Lum Corp. of America v. Alcoa Bldg. Prods. Inc. (1976) ALCOA canceled an exclusive distributorship agreement after 8 years with Bak-

A-Lum after it knew that a new warehouse lease had been entered into by Bak. Bak sued for failure to give notice and keeping plans secret.

Notice of termination of an exclusive arrangement must be meaningful and must be designed to minimize the other party’s loss.

Page 48: Contracts Outline Burton Case Book CUA Law 2011

RULE: An implied requirement of good-faith dealings is implied in franchise/exclusive dealings agreements, and its breach is actionable.

Badgett v. Security State Bank “in and out of the dairy industry” Badgetts contended that Security State Bank had breached the duty of good faith

by not considering their terms for another loan renegotiation. Long history of prior dealings.

Unclear if holding is good law, vs. appellate holding to the contrary.RULE: A failure to consider proposed renegotiation terms cannot serve as the basis for a breach of duty of good faith.

J.J. Brooksbank Co. v. Budget Rent-A-Car (1983) “telephone reservation system” Budget contended it was no longer obligated to give Brooksbank free reservations

because technological advancements had rendered the original K impracticable. Majority: The only event in this case which occurred was the normal progression

of technology. While clearly this was not specifically anticipated, it does not go to the essence of the K. A reduction rather than elimination of the fees is the equitable resolution.

Dissent: in 1962 the parties didn’t foresee the tech changes made by budget in 1970, thus “cost-free reservations” clause in K is irrelevant. Brooksbank does deserve a reduction. But the proper way is to base it on the equities of the parties entire business relationship, a percentage…

RULE: Impracticability will excuse performance of a K only where an event occurs, the nonoccurrence of which was a basic assumption of the K at the time it was made.

NOTE on Equitable Adjustment – equity dispenses with rules and K terms in favor of doing particularized justice, where judges have discretion.

Sec. 3. Tortious Conduct in Contract Performance NOTE on “Contorts” – is breach of the implied covenant of good faith and fair

dealing, particularly in insurance Ks, a tort, for which emotional distress and punitive damages might be available? Consider conditions under which tort damages should be available for a breach of K that is not an independent tort.

Gruenberg v. Aetna Insurance Co. (1973) “Fire at the Brass Rail” More open to contorts than Beck. Gruenberg contended that his property damage insurers breached the duty of good

faith by encouraging a criminal investigation they knew to be unwarranted.RULE: An insurer violates the duty of good faith by encouraging a criminal investigation of its insured that it knows to be unwarranted.

Beck v. Farmers Insurance Exchange (1985) “hit and run, insurer failed to negotiate” RULE: An insurer breaches its duty of good faith by unreasonably failing to investigate and negotiate.

Page 49: Contracts Outline Burton Case Book CUA Law 2011

Ch. 8. Rights of Third Parties

Sec. 1. Third Party Beneficiaries Doctrine of Privity, which held that only parties to a K could bring actions to

enforce them, is receding. Limited the exposure of a promisor to unforeseeable and sometimes crushing liabilities.

Lawrence v. Fox (1859) “creditor beneficiary” Holly(promissee/debtor) owed L(creditor ben/3PB) $. H loaned $ to

F(promisor/debtor); F as consideration gave H promise to pay her debt to L. F didn’t pay L. L sues on F’s breach of promise to H even though lacked privity.

Ultimately majority allows L to recover because it was manifestly “just”. Dissent (Comstock): Privity is essential. L had nothing to do with the promise;

was not made to him, nor did the consideration proceed from him. Presumably, he should sue H, who would in turn need to sue F. Worried about freedom of K

RULE: A third party for whose benefit a K is made may bring an action for its breach

Seaver v. Ransom (1918) “donee beneficiary” Scum-bag Judge Berman made a promise to wife on her deathbed to leave the

house to their niece, Seaver, who sued JBerman’s executor for breach when he died and didn’t leave her anything. (statute of frauds would keep oral promise out, right?)

Turned on “family” exception, but again really had to do with “justice” and intent of parties.

RULE: A niece for whose benefit a promise was made to her aunt may successfully bring an action for breach of that promise.

Restatement FIRST of Contracts § 133 – Def. of Donee Beneficiary, Creditor Beneficiary, Incidental Beneficiary

o 3rd party is a donee beneficiary if it appears from the terms and circumstances that the purpose of the promisee in obtaining the promise is to make a gift to the beneficiary or confer upon him a right against the promisor to some performance not due from the promisee to the beneficiary;

o 3rd party is a creditor beneficiary if no purpose to make a gift appears from the terms or circumstances and performance of the promise will satisfy an actual or supposed or asserted duty of the promisee to the beneficiary…

o 3rd party is an incidental beneficiary if neither of above true. R § 302 – Intended and Incidental Beneficiaries

o Either (a) the performance of the promise will satisfy an obligation of the promisee to pay the beneficiary (“creditor beneficiary”) (Lawrence); or

o (b) the circumstances indicate that the promisee intends to give the beneficiary the benefit of the promised performance(“donee ben”)(Seaver)

o All others are incidental beneficiaries.

Page 50: Contracts Outline Burton Case Book CUA Law 2011

R § 304 – intended beneficiary may enforce promisor’s duty created by promise

R § 315 – incidental beneficiary acquires no right against promisee/p’or

Bain v. Gillispie (1984) “basketball referee v. novelty store owner. Example of new restatement collapsing distinction into “intended beneficiary” but

relying all the same on the two distinct notions. Bain (referee) and Big 10 League did not intend gift to Gillispie and clearly not a

creditor benefitiary either.RULE: The real test as to whether a party is a beneficiary under a K is whether the contracting parties intended that a third person should receive a benefit which might be enforced in the courts.

Lonsdale v. Chesterfield (1983) Complicated fact pattern – I give up! How to interpret intent of parties to confer rights or delegate duties to a 3rd PB

RULE: The parties must have intended that the promisor assume a direct obligation to the intended beneficiary at the time they entered into the K in order to create a third-party beneficiary.

Cretex Companies, Inc. v. Construction Leaders, Inc. (1984) Performance bond guaranteeing principles performance, not guaranteeing

payment, which can be guaranteed through payment bonds. Cretex would have had greater chance of success with latter. Perf bond only guarnatee’s the principal’s (GC) performance to the obligee (owner/developer/Northland Mortgage). Cretex would have had to be an express 3PB but was not.

Dissent: Traveler’s (surety) obligation to “faithfully perform the K” included the payment for all sub K’rs labor…

RULE: Third-party beneficiaries may only enforce Ks if they were an intended beneficiary of the agreement.

R § 313 – Gov’t Ks – limits 3rd PBs to prevent general population claiming rights against Gov’t contractors.

Martinez v. Socoma Companies, Inc. (1974) BURKE (dissent) “low-income job project” See Waterman Lake Socoma (promisor) contracted with US (promisee) to lease spaces for the US to

establish low income jobs for people in community like Martinez (3PB). US paid but Socoma failed to perform. Martinez sues (class action).

Majority: Martinez was neither creditor ben (person on whom promisor’s performance of the K will discharge some form of legal duty owed to the veneficiary by the promisee – Govt at no time bore any legal duty toward M) nor donee ben (for whom the contractual intent is either to make a gift or confer on them a right against the promisor – govt did not intend to make a figt nor to confer on M a right against S). Finally, that the K confers benefits on M without M giving consideration doesn’t mean the benefits are intended as gifts; rather they’re means to the end of the policy – ie improving ‘hood/greater social good.

Page 51: Contracts Outline Burton Case Book CUA Law 2011

Burke dissent: M was express, not incidental ben. Benefits conferred on him were not simply means, also ends. He could have proved he was donee ben.

RULE: Only creditor beneficiaries and donee beneficiaries qualify as third-party beneficiaries and have enforceable rights under Ks to which they are not parties.

R § 309 – Defenses Against the Beneficiary R § 311 – Variation of a Duty to a Beneficiary UCC § 2-318 – 3rd Party Beneficiaries of Warranties Express or Implied

Rouse v. U.S. (1954) “defenses against 3PB available to promisor” Bessie=promisee; Rouse=promisor; US=3PB. Rouse promised to pay Bessie, Winston’s creditor, but refused after discovering

flaws in his own K with Bessie and in Bessie’s K with her creditor. US sued Rouse (3PB suing promisor) as 3PB of R’s K with B.

RULE (1): A promisor may assert against a 3PB a defense that he would have against the promisee.

RULE (2): But a promisor may not assert against a 3PB a defense which the promisee would have against the beneficiary.

RULE: A third-party beneficiary’s rights against the promisor rise no higher than those of the promisee; a 3PBs rights may rise higher against the promisor than they could against the promisee.

U.S. v. Wood (1989)RULE: Third parties may enforce Ks if they are the intended beneficiaries of a valid agreement.

Sec. 2. Assignment and Delegation Assign rights; delegate duties. an assignment is the present transfer of a K right

from one person to another. It is an executed rather than an executory transaction (unlike a K). Transaction based rather than K based. UCC Article 9 covers much of it (secured transactions). Thus closer to a sale of goods. Free alienability informs the law, but as always courts balance it against autonomy and security principles.

Adams v. Merced Stone Co. (1917)RULE: a gratuitous transfer of the right to collect a debt is not valid absent delivery.

UCC §§ 9-404 (rights acquired by assignee; claims and defenses against assignee), 9-406 (discharge of account debtor; notification of assignment;…restrictions)

Ertel v. Radio Corp. of America (1974) Constructive notice

RULE: The subrogee of a creditor takes the debt subject to applicable setoffs.

R § 318 / UCC § 2-210 – Delegation of Performance of Duty

Page 52: Contracts Outline Burton Case Book CUA Law 2011

Crane Ice Cream Co. v. Terminal Freezing & Heating Co. Crane – assignee v. Terminal-seller. Terminal refused to perform a K when

Frederick-assignor attempted to assign its rights and delegate its duties to Crane. Frederick-assignor-ice cream manufacturer, entered into K with Terminal-seller,

buying ice from terminal in return for promise to sell all ice he needed up to 250 tons/week. No minimum guaranty. Personal relationship. F assigned rights and duties to Crane-assignee-large ice cream manufacturer. Terminal terminated. Crane sued for breach of K, asserting third party rights. Court holds (see rule).

Crane was a large corp. without personal relationship with Terminal. F as owner of sole company was obliged to purchase all ice from T. K depended on relationship. Only where an assignee would provide the same performance is assignment ok.

Case highlights the conflict that arises between autonomy of parties and freedom of K in assignment cases. Unlike Ks for sale of goods, the assignment deals with obligations various parties will have toward one another and how those obligations can shift.

RULE: A party may not delegate its duties under a K unless performance by another is substantially the same.

British Waggon Co. & Parkgate Waggon Co. v. Lea & Co. (1880 England) Parkgate-assignor entered into K with Lea-merchants to provide wagons for hire

and repair service on them. Then Parkgate dissolved and transferred liabilities and assest. Ks were assigned to British Waggon Co-assignee. Lea then declared it was no longer bound by Ks as its agreement had been with Parkgate, not British.

Typically, rule is Ks are assignable. When a nonpersonal service K (personal service Ks typically involve unique services like entertainers) is assigned, it is enforceable. Here the K called only for competent repair and maintenance.

RULE: when a non-personal service K is assigned, it remains enforceable

Sally Beauty Co., Inc. v. Nexxus Prods. Co., Inc. (1986) POSNER (dissent) Nexxus (manufacturer) entered K with Best under which Best would be exclusive

distributor of Nexxus products. Sally acquired Bess in a stock purchase, and succeeded to Best’s rights on all Ks, including with N. Sally was wholly owned by a direct competitor of N. N refused to allow S to be the distributor and canceled K. S sued N for breach. Trial ct summary judgment for N on grounds that K was one for personal services and thus not assignable to S.

Majority agreed for diff reason – UCC 2-210 provides that “a party may perform his duty through a delegate unless otherwise agreed to or unless the other party has a substantial interest in having his original promisor perform or control the acts required by the K.” N had substantial interest in not having a subsidiary of its direct competitor be exclusive dealer of its products. S could not guarantee the same efforts as Best could under the circumstances. Thus N had the right to reject assignment of the K to S.

POSNER: K issue was not a personal services K and should not have been decided on summary judgment.

Page 53: Contracts Outline Burton Case Book CUA Law 2011

RULE: the duty of performance under an exclusive distributorship may not be delegated to a direct competitor without the obligee’s consent.